Buy BOOKS at Discounted Price

Arithmetic Progressions (ap)

Class 10th Mathematics KC Sinha Solution
Exercise 8.1
  1. tn = 3n + 1 Write the first three terms of the following sequences defined by:…
  2. t_n = 2^n Write the first three terms of the following sequences defined by:…
  3. tn = n^2 + 1 Write the first three terms of the following sequences defined by:…
  4. tn = n(n + 2) Write the first three terms of the following sequences defined…
  5. tn = 2n+5 Write the first three terms of the following sequences defined by:…
  6. t_n = n-3/4 Write the first three terms of the following sequences defined by:…
  7. t_n = n (n-2)/n+3 t_20 Find the indicated terms in each of the following…
  8. t_n = n (n-2)/n+3 t_20 Find the indicated terms in each of the following…
  9. t_n = (n - 1)(2 - n)(3 + n) ; t20 Find the indicated terms in each of the…
  10. t_n = t_n-1/n^2 , t_1 = 3 t_2 , t_3 , (n geater than or equal to 2) Find the…
  11. t_1 = 3 , t_n = 3t_n-1+2 Write the next three terms of the following sequences:…
  12. t_1 = 3 , t_n = 3t_n-1+2 for all n1 Write the next three terms of the following…
  13. a = 1, d = 1 Write the first four terms of the A.P. when first term a and…
  14. a= 3, d=0 Write the first four terms of the A.P. when first term a and common…
  15. a = 10, d = 10 Write the first four terms of the A.P. when first term a and…
  16. a= -2, d=0 Write the first four terms of the A.P. when first term a and common…
  17. a = 100, d = -30 Write the first four terms of the A.P. when first term a and…
  18. a= -1, d= 1/2 Write the first four terms of the A.P. when first term a and…
  19. a = -7, d = -7 Write the first four terms of the A.P. when first term a and…
  20. a = 1, d = 0.1 Write the first four terms of the A.P. when first term a and…
  21. 6, 3, 0, - 3, ... For the following A.P's write the first term and common…
  22. - 3.1, - 3.0, - 2.9, - 2.8, ... For the following A.P's write the first term…
  23. 147, 148, 149, 150, ... For the following A.P's write the first term and common…
  24. - 5, - 1, 3, 7, ... For the following A.P's write the first term and common…
  25. 3, 1, - 1, - 3, ... For the following A.P's write the first term and common…
  26. 2 , 2 1/3 , 2 2/3 ,-3 For the following A.P's write the first term and common…
  27. 3/2 , 1/2 , - 1/2 , - 3/2 , l For the following A.P's write the first term and…
  28. 1, - 1, - 3, - 5, Which of the following list of numbers form A.R's? If they…
  29. 2, 4, 8, 16, ... Which of the following list of numbers form A.R's? If they…
  30. - 2, 2, - 2, 2, - 2, ... Which of the following list of numbers form A.R's? If…
  31. - 1/2 , - 1/2 , - 1/2 , - 1/2 , l Which of the following list of numbers form…
  32. 2 , 5/2 , 3 7/2 , l l Which of the following list of numbers form A.R's? If…
  33. 0, - 4, - 8, - 12, Which of the following list of numbers form A.R's? If they…
  34. 4, 10, 16, 22, ... Which of the following list of numbers form A.R's? If they…
  35. a, 2a, 3a, 4a, ... Which of the following list of numbers form A.R's? If they…
  36. - 1.2, - 3.2, - 5.2, - 7.2, ... Which of the following list of numbers form…
  37. root 3 , root 12 , root 48 , root 192 Which of the following list of numbers…
  38. a, a^2 , a^3 , a^4 , ….. Which of the following list of numbers form A.R's? If…
  39. 1, 3, 9, 27, ... Which of the following list of numbers form A.R's? If they…
  40. 1^2 , 2^2 , 3^2 , 4^2 , … Which of the following list of numbers form A.R's? If…
  41. 1^2 , 5^2 , 7^2 , 7^2 , … Which of the following list of numbers form A.R's? If…
  42. 1^2 , 3^2 , 5^2 , 7^2 , … Which of the following list of numbers form A.R's? If…
  43. The salary of a teacher in successive years when starting salary is Rs. 8000,…
  44. The taxi fare after each km when the fare is Rs. 15 for the first km and Rs. 8…
  45. The lengths of the rungs of a ladder when the bottom rung is 45cm, and length…
  46. The amount of money in the account every year when Rs. 10000 is deposited at…
  47. The money saved by Sudha in successive years when she saves Rs. 100 the first…
  48. Number of pairs of rabbits in successive months when the pair of rabbits is too…
  49. In which of the following situations does the list of numbers involved…
Exercise 8.2
  1. 1, 6, 11, 16, ..., t16, Find the indicated terms in each of the following…
  2. a = 3, d = 2; ; tn, t10 Find the indicated terms in each of the following…
  3. — 3, — 1/2, 2, ... ; t10, Find the indicated terms in each of the following…
  4. a = 21, d = — 5; tn, t25 Find the indicated terms in each of the following…
  5. Find the 10th term of the A.P. 10, 5, 0, — 5, — 10, ...
  6. Find the 10th term of the A.P. 13/5 , 7/5 , 1/5 ,-1 , l
  7. Find the sum of 20th and 25th terms of A.P. 2, 5, 8, 11, ...
  8. 6, 3, 0, — 3,…..,-36 Find the number of terms in the following A.P.'s…
  9. 5/6 , 1 , 1 1/6 , l , 3 1/3 Find the number of terms in the following A.P.'s…
  10. Determine the number of terms in the A.P. 3, 7, 11, ..., 399. Also, find its…
  11. Which term of the A.P. 5, 9, 13, 17, ... is 81?
  12. Which term of the A.P. 14, 9, 4, - I, - 6, ... is - 41 ?
  13. Which term of A.P. 3, 8, 13, 18, ... is 88?
  14. Which term of A.P. 5/6 , 1 , 1 1/6 , 1 1/3 , l is 3?
  15. Which term of A.P. 3, 8, 13, 18, ..., is 248 ?
  16. Find the 6th term from end of the A.P. 17, 14, 11,… - 40.
  17. Find the 8th term from end of the A.P. 7, 10, 13, ..., 184.
  18. 6, 10, 14, 18, ..., 174? Find the number of terms of the A.P.
  19. 7, 11, 15, ..., 139? Find the number of terms of the A.P.
  20. 41, 38, 35, ..., 8? Find the number of terms of the A.P.
  21. Find the first negative term of sequence 999, 995, 991, 987, ...
  22. Is 51 a term of the A.P. 5, 8, 11, 14, ...?
  23. Is 56 a term of the A.P. 4 , 4 1/2 , 5 , 5 1/2 , 6 , l ?
  24. The 7th term of an A.P. is 20 and its 13th term is 32. Find the A.P. [CBSE…
  25. The 7th term of an A.P. is - 4 and its 13th term is - 16. Find the A.P. [CBSE…
  26. The 8th term of an A.P. is 37, and its 12th term is 57. Find the A.P.…
  27. Find the 10th term of the A.P. whose 7th and 12th terms are 34 and 64…
  28. 13, 19, 25, ... and 69, 68, 67, ... For what value of n are the nth term of…
  29. 23, 25, 27, 29, ... and - 17, - 10, - 3, 4, ... For what value of n are the…
  30. 24, 20, 16, 12, ... and - 11, - 8, - 5, - 2, ... For what value of n are the…
  31. 63, 65, 67, ... and 3, 10, 17, ... For what value of n are the nth term of the…
  32. 5, □,□, 9 1/2 In the following A.P., find the missing terms:
  33. 54, □,□, 42 In the following A.P., find the missing terms:
  34. - 4, □,□,□,□, 6 In the following A.P., find the missing terms:
  35. □, 13, □, 3 In the following A.P., find the missing terms:
  36. 7, □,□,□,27 In the following A.P., find the missing terms:
  37. 2, □, 26 In the following A.P., find the missing terms:
  38. □, □, 13, □, □, 22 In the following A.P., find the missing terms:…
  39. - 4, □, □, □, 6 In the following A.P., find the missing terms:
  40. □, 38, □,□,□, - 22 In the following A.P., find the missing terms:…
  41. If 10th term of an A.P. is 52 and 17th term is 20 more than the 13th term,…
  42. Which term of the A.P. 3, 15, 27, 39, ... will be 132 more than its 54th term?…
  43. Which term of the A.P. 3, 10, 17, 24, ... will be 84 more than its 13th term ?…
  44. The 4th term of an A.P. is zero. Prove that its 25th term is triple its 11th…
  45. If 10 times the 10th term of an A.P. is equal to 15 times the 15th term, show…
  46. If (m + 1)th term of an A.P. is twice the (n + 1)th term, prove that (3m + 1)th…
  47. If tn be the nth term of an A.P. such that t_4/t_7 = 2/3 find t_8/t_9 .…
  48. Find the number of all positive integers of 3 digits which are divisible by 5.…
  49. How many three digit numbers are divisible by 7.
  50. If tn denotes the nth term of an A.P., show that tm + t2n+m = 2 tm+n.…
  51. Find a if 5a + 2, 4a - I, a + 2 are in A.P.
  52. nth term of a sequence is 2n + 1. Is this sequence an A.P.? If so find its…
  53. The sum of the 4th and Sth terms of an A.P. is 24 and the sum of the 6th and…
  54. A person was appointed in the pay scale of Rs. 700-40-1500. Find in how many…
  55. A sum of money kept in a hank amounts to Rs. 600/- in 4 years and Rs. 800/- in…
  56. A man starts repaying, a loan with the first instalment of Rs. 100. If he…
Exercise 8.3
  1. Three numbers are in A.P. Their sum is 27 and the sum of their squares is 275.…
  2. The sum of three numbers in A.P. is 12 and the sum of their cubes is 408. Find…
  3. Divide 15 into three parts which are in A.P. and the sum of their squares is…
  4. Divide 20 into four parts which are in A.P. such that the ratio of the product…
  5. Sum of three numbers in A.P. is 21 and their product is 231. Find the numbers.…
  6. Sum of three numbers in A.P. is 3 and their product is — 35. Find the numbers.…
  7. If a/b+c , b/c+a , c/a+b are in A.P. and a + b + c ≠ 0, prove that 1/b+c , 1/c+a…
  8. If a^2 , b^2 , c^2 are in A.P., show that a/b+c , b/c+a , c/a+b are in A.P.…
  9. 1/bc , 1/ca , 1/ab are in A.P. If a, b, c are in A.P., prove that…
  10. (b + c)^2 — a^2 , (c + a)^2 — b^2 , (a + b)^2 — c^2 are in A.P. If a, b, c are…
  11. 1/root b + root c , 1/root c + root a , 1/root a + root b are in A.P. If a, b,…
  12. b+c-a/a , c+a-b/b , a+b-c/c are in A.P., show that 1/a , 1/b , 1/c are in A.P.…
  13. If (b - c)^2 , (c - a)^2 , (a - b)^2 are in A.P., then show that: 1/b-c , 1/c-a…
  14. (a — c)^2 = 4 (a — b)(b — c) If a, b, c are in A.P., prove that:
  15. a^3 + c^3 + 6abc = 8b^3 If a, b, c are in A.P., prove that:
  16. (a + 2b — c)(2b + c — a)(c + a — b) = 4abc [Hint: Put b = a+c/2 on L.H.S. and…
Exercise 8.4
  1. The sum of n terms of an A.P. is (5n^2/2 + 3n/2) . Find its 20th term.…
  2. The sum of first n terms of an A.P. is given by Sn = 3n^2 + 2n. Determine the…
  3. The sum of the first n terms of an A.P. is given by Sn = 2n^2 + 5n , find the…
  4. The sum of n terms of an A.P. is 3n^2 + 5n. Find the A.P. Hence, find its 16th…
  5. If the sum of the first n terms of an A.P. is given by Sn = (3n^2 - n), find its…
  6. If the sum to first n terms of an A.P. is (3n^2/2 + 5n/2) , find its 25th term.…
  7. If the nth term of an A.P. is (2n + 1), find the sum of first n terms of the…
  8. If the nth term of an A.P. is 9 — 5n, find the sum to first 15 terms.…
  9. Find the sum of first 25 terms of an A.P. whose nth term is 1 — 4n.…
  10. If the sum to n terms of a sequence be n^2 + 2n, then prove that the sequence is…
  11. Find the sum to first n terms of an A.P. whose kth term is 5k + 1.…
  12. If the sum of n terms of an A.P. is 3n^2 + 5n and its mth term is 164, find the…
  13. If the sum of n terms of an A.P. is pn + qn^2 , where p and q are constants,…
  14. If the sum of n terms of an A.P. is nP + 1/2 n(n —1)Q , where P and Q are…
  15. Find the sum : 25 + 28 + 31 +… + 100
  16. Which term of the A.P. 4, 9, 14, ... is 89? Also, find the sum 4 + 9 + 14 + +…
  17. 1 + 6+11 + 16 +...+x= 148 Solve for x
  18. 25+22+19+ 16+...+x= 115 Solve for x
  19. Find the number of terms of the A.P. 64, 60, 56, ... so that their sum is 544.…
  20. How many terms of the A.P. 3, 5, 7, 9, ... must be added to get the sum 120?…
  21. Find the number of terms of the A.P. 63, 60, 57, ... so that their sum is 693.…
  22. How many terms of the series 15 + 12 + 9 + ... must be taken to make 15?…
  23. Find the sum of all the odd numbers lying between 100 and 200.
  24. Find the sum of all odd integers from 1 to 2001.
  25. Determine the sum of first 35 terms of an A.P., if the second term is 2 and the…
  26. If the sum of the first p terms of an A.P. is q and the sum of first q terms is…
  27. How many terms of the A.P. -6,- 11/2 ,-5 ... are needed to get the sum - 25?…
  28. Find the sum of the numbers lying between 107 and 253 that are multiples of 5.…
  29. Find the sum of all natural numbers lying between 100 and 1000 which are…
  30. Find the sum of all the two digit odd positive integers.
  31. Find the sum of all multiplies of 9 lying between 300 and 700.
  32. Find the sum of all the three digit natural numbers which are multiples of 7.…
  33. Find the sum of all natural numbers lying between 100 and 500, which are…
  34. Find the sum of all the 3 digit natural numbers which are divisible by 13.…
  35. The 5th and 15th terms of an A.P. are 13 and - 17 respectively. Find the sum of…
  36. Find the sum of first 21 terms of the A.P. whose 2nd term is 8 and 4th term is…
  37. Find the sum of 51 terms of the A.P. whose second term is 2 and the 4th term is…
  38. Find the sum of the first 25 terms of the A.P. whose 2nd term is 9 and 4th term…
  39. If the sum of 8 terms of an A.P. is 64 and the sum of 19 terms is 361, find…
  40. The first and the last terms of an A.P. are 17 and 350 respectively. If the…
  41. If a, b, c be the 1st, 3rd and nth terms respectively of an A.P., there prove…
  42. If the mth term of an A.P. is 1/n and the nth term is 1/m , then prove that the…
  43. If the 12th term of an A.P. is - 13 and the sum of the first four terms is 24,…
  44. If the number of terms of an A.P. be 2n + 3, then find the ratio of sum of the…
  45. If the sum of first m terms of an A.P. is the same as the sum of its first n…
  46. In an A.P. the first term is 2, and the sum of the first five terms is…
  47. If d be the common difference of an A.P. and Sn be the sum of its n terms, then…
  48. The sum of the first 7 terms of an A.P. is 10, and that of the next 7 terms is…
  49. If the pth term of an A.P. is x and qth term is y, show that the sum of (p + q)…
  50. The sum of 17 terms of two series in A.P. are in the ratio (3n + 8) : (7n +…
  51. The sum of 11 terms of two A.P.'s are in the ratio (5n + 4) : (9n + 6), find…
  52. In an A.P. Sn denotes the sum to first n terms, if Sn = n^2 p and Sm = m^2 p (m…
  53. The income of a person is Rs. 300000 in the first year and he receives an…
  54. A man starts repaying a loan as first installment of Rs. 100. If he increases…
  55. The interior angles of a polygon are in A.P., the smallest angle is 75° and the…

Exercise 8.1
Question 1.

Write the first three terms of the following sequences defined by:

tn = 3n + 1


Answer:

Given: tn = 3n + 1


Taking n = 1, we get


t1 = 3(1) + 1 = 3 + 1 = 4


Taking n = 2, we get


t2 = 3(2) + 1 = 6 + 1 = 7


Taking n = 3, we get


t3 = 3(3) + 1 = 9 + 1 = 10


Hence, the first three terms are 4, 7 and 10.



Question 2.

Write the first three terms of the following sequences defined by:



Answer:

Given: tn = 2n


Taking n = 1, we get


t1 = 21 = 2


Taking n = 2, we get


t2 = 22 = 2 × 2 = 4


Taking n = 3, we get


t3 = 23 = 2 × 2 × 2 = 8


Hence, the first three terms are 2, 4 and 8.



Question 3.

Write the first three terms of the following sequences defined by:

tn = n2 + 1


Answer:

Given: tn = n2 + 1


Taking n = 1, we get


t1 = (1)2 + 1 = 1 + 1 = 2


Taking n = 2, we get


t2 = (2)2 + 1 = 4 + 1 = 5


Taking n = 3, we get


t3 = (3)2 + 1 = 9 + 1 = 10


Hence, the first three terms are 2, 5 and 10.



Question 4.

Write the first three terms of the following sequences defined by:

tn = n(n + 2)


Answer:

Given: tn = n(n+2)


Taking n = 1, we get


t1 = (1)(1+2) = (1)(3) = 3


Taking n = 2, we get


t2 = (2)(2+2) = (2)(4) = 8


Taking n = 3, we get


t3 = (3)(3+2) = (3)(5) = 15


Hence, the first three terms are 3, 8 and 15.



Question 5.

Write the first three terms of the following sequences defined by:

tn = 2n+5


Answer:

Given: tn = 2n + 5


Taking n = 1, we get


t1 = 2(1) + 5 = 2 + 5 = 7


Taking n = 2, we get


t2 = 2(2) + 5 = 4 + 5 = 9


Taking n = 3, we get


t3 = 2(3) + 5 = 6 + 5 = 11


Hence, the first three terms are 7, 9 and 11.



Question 6.

Write the first three terms of the following sequences defined by:



Answer:

Given:


Taking n = 1, we get



Taking n = 2, we get



Taking n = 3, we get



Hence, the first three terms are



Question 7.

Find the indicated terms in each of the following sequence whose nth terms are:



Answer:

Given:


Now, we have to find t1 and t2.


So, in t1, n = 1



Now, t2, n = 2




Question 8.

Find the indicated terms in each of the following sequence whose nth terms are:



Answer:

Given:


So,



Question 9.

Find the indicated terms in each of the following sequence whose nth terms are:

= (n - 1)(2 - n)(3 + n) ; t20


Answer:

Given: tn = (n – 1)(2 – n)(3+n)


So, t20 = (20 – 1)(2 – 20)(3+20)


= (19)(-18)(23)


= -7866



Question 10.

Find the indicated terms in each of the following sequence whose nth terms are:



Answer:

Given:


So, [given: t1 = 3]


and



Question 11.

Write the next three terms of the following sequences:



Answer:

Given: t2 = 2 and tn = tn-1 + 1


Now, we have to find next three terms i.e. t3, t4 and t5


Taking n = 3, we get


t3 = t3-1 + 1


= t2 + 1


= 2 + 1 [given: t2 = 2]


t3 = 3 …(i)


Taking n = 4, we get


t4 = t4-1 + 1


= t3 + 1


= 3 + 1 [from (i)]


t4 = 4 …(ii)


Taking n = 5, we get


t5 = t5-1 + 1


= t4 + 1


= 4 +1


t5 = 5 [from (ii)]


Hence, the next three terms are 3, 4 and 5.



Question 12.

Write the next three terms of the following sequences:

for all


Answer:

Given: t1 = 3 and tn = 3tn-1 + 2


Now, we have to find next three terms i.e. t2, t3 and t4


Taking n = 2, we get


t2 = 3t2-1 + 2


= 3t1 + 2


= 3(3) + 2 [given: t1 = 3]


t3 = 9 + 2


t3 = 11 …(i)


Taking n = 3, we get


t3 = 3t3-1 + 2


= 3t2 + 2


= 3(11) + 2 [from (i)]


= 33 + 2


t3 = 35 …(ii)


Taking n = 4, we get


t4 = 3t4-1 + 2


= 3t3 + 2


= 3(35) +2


t5 = 105 + 2


t5 = 107 [from (ii)]


Hence, the next three terms are 11, 35 and 107.



Question 13.

Write the first four terms of the A.P. when first term a and common difference d are given as follows:

a = 1, d = 1


Answer:

Given: a = 1 and d = 1


The general form of an A.P is a, a+d, a+2d, a+3d,…


So, the first term a is 1 and the common difference d is 1, then the first four terms of the AP is


1, (1+1), (1+2×1), (1+3×1)


⇒ 1, 2, 3, 4


Hence, the first four terms of the A.P. is 1, 2, 3 and 4.



Question 14.

Write the first four terms of the A.P. when first term a and common difference d are given as follows:

a= 3, d=0


Answer:

Given: a = 3 and d = 0


The general form of an A.P is a, a+d, a+2d, a+3d,…


So, the first term a is 3 and the common difference d is 0, then the first four terms of the AP is


3, (3+0), (3+2×0), (3+3×0)


⇒ 3, 3, 3, 3


Hence, first four terms of the A.P. is 3, 3, 3 and 3.



Question 15.

Write the first four terms of the A.P. when first term a and common difference d are given as follows:

a = 10, d = 10


Answer:

Given: a = 10 and d = 10


The general form of an A.P is a, a+d, a+2d, a+3d,…


So, the first term a is 10 and the common difference d is 10, then the first four terms of the AP is


10, (10+10), (10+2×10), (10+3×10)


⇒ 10, (20), (10+20), (10+30)


⇒ 10, 20, 30, 40


Hence, first four terms of the A.P. is 10, 20, 30 and 40.



Question 16.

Write the first four terms of the A.P. when first term a and common difference d are given as follows:

a= -2, d=0


Answer:

Given: a = -2 and d = 0


The general form of an A.P is a, a+d, a+2d, a+3d,…


So, the first term a is -2 and the common difference d is 0, then the first four terms of the AP is


-2, (-2+0), (-2+2×0), (-2+3×0)


⇒ -2, -2, -2, -2


Hence, the first four terms of the A.P. is -2, -2, -2 and -2.



Question 17.

Write the first four terms of the A.P. when first term a and common difference d are given as follows:

a = 100, d = -30


Answer:

Given: a = 100 and d = -30


The general form of an A.P is a, a+d, a+2d, a+3d,…


So, the first term a is 100 and the common difference d is -30, then the first four terms of the AP is


100, (100+(-30)), (100+2×(-30)),(100+3×(-30))


⇒ 100, (100 – 30), (100 – 60), (100 – 90)


⇒ 100, 70, 40, 10


Hence, first four terms of the A.P. is 100, 70, 40 and 10.



Question 18.

Write the first four terms of the A.P. when first term a and common difference d are given as follows:

a= -1, d= 1/2


Answer:

Given: a = -1 and d


The general form of an A.P is a, a+d, a+2d, a+3d,…


So, the first term a is 1 and the common difference d is , then the first four terms of the AP is





Hence, first four terms of the A.P. is .



Question 19.

Write the first four terms of the A.P. when first term a and common difference d are given as follows:

a = -7, d = -7


Answer:

Given: a = -7 and d = -7


The general form of an A.P is a, a+d, a+2d, a+3d,…


So, the first term a is -7, and the common difference d is -7, then the first four terms of the AP is


-7, (-7+(-7)), (-7+2×(-7)), (-7+3×(-7))


⇒ -7, (-7 – 7), (-7 – 14), (-7 – 21)


⇒ -7, -14, -21, -28


Hence, the first four terms of the A.P. is -7, -14, -21 and -28.



Question 20.

Write the first four terms of the A.P. when first term a and common difference d are given as follows:

a = 1, d = 0.1


Answer:

Given: a = 1 and d = 0.1


The general form of an A.P is a, a+d, a+2d, a+3d,…


So, the first term a is 1, and the common difference d is 0.1, then the first four terms of the AP is


1, (1+0.1), (1+2×(0.1)), (1+3×(0.1))


⇒ 1, 1.1, 1.2, 1.3


Hence, the first four terms of the A.P. is 1, 1.1, 1.2 and 1.3.



Question 21.

For the following A.P's write the first term and common difference:

6, 3, 0, - 3, ...


Answer:

In general, for an AP a1, a2, . . . .,an, we have


d = ak+1 - ak


where ak+1 and ak are the (k+1)th and kth terms respectively.


For the list of numbers: 6, 3, 0, -3, . . .


a2 – a1 = 3 – 6 = -3


a3 – a2 = 0 – 3 = -3


a4 – a3 =-3 – 0 = -3


Here, the difference of any two consecutive terms in each case is -3.


So, the given list is an AP whose first term a is 6, and common difference d is -3.



Question 22.

For the following A.P's write the first term and common difference:

- 3.1, - 3.0, - 2.9, - 2.8, ...


Answer:

In general, for an AP a1, a2, . . . .,an, we have


d = ak+1 - ak


where ak+1 and ak are the (k+1)th and kth terms respectively.


For the list of numbers: - 3.1, - 3.0, - 2.9, - 2.8, ...


a2 – a1 = -3.0 – (-3.1) = -3.0 + 3.1 = 0.1


a3 – a2 = -2.9 – (-3.0) = -2.9 + 3.0 = 0.1


a4 – a3 = -2.8 – (-2.9) = -2.8 + 2.9 = 0.1


Here, the difference of any two consecutive terms in each case is 0.1. So, the given list is an AP whose first term a is -3.1 and common difference d is 0.1.



Question 23.

For the following A.P's write the first term and common difference:

147, 148, 149, 150, ...


Answer:

In general, for an AP a1, a2, . . . .,an, we have


d = ak+1 - ak


where ak+1 and ak are the (k+1)th and kth terms respectively.


For the list of numbers: 147, 148, 149, 150, ...


a2 – a1 = 148 – 147 = 1


a3 – a2 = 149 – 148 = 1


a4 – a3 = 150 – 149 = 1


Here, the difference of any two consecutive terms in each case is -1. So, the given list is an AP whose first term a is 147 and common difference d is 1.



Question 24.

For the following A.P's write the first term and common difference:

- 5, - 1, 3, 7, ...


Answer:

In general, for an AP a1, a2, . . . .,an, we have


d = ak+1 - ak


where ak+1 and ak are the (k+1)th and kth terms respectively.


For the list of numbers: - 5, - 1, 3, 7, ...


a2 – a1 = -1 – (-5) = -1 + 5 = 4


a3 – a2 = 3 – (-1) = 3 + 1 = 4


a4 – a3 = 7 – 3 = 4


Here, the difference of any two consecutive terms in each case is -4. So, the given list is an AP whose first term a is -5 and common difference d is 4.



Question 25.

For the following A.P's write the first term and common difference:

3, 1, - 1, - 3, ...


Answer:

In general, for an AP a1, a2, . . . .,an, we have


d = ak+1 - ak


where ak+1 and ak are the (k+1)th and kth terms respectively.


For the list of numbers: 3, 1, - 1, - 3, ...


a2 – a1 = 1 – 3 = -2


a3 – a2 =-1 – 1 = -1 - 1 = -2


a4 – a3 =-3 – (-1) = -3 + 1= -2


Here, the difference of any two consecutive terms in each case is --2. So, the given list is an AP whose first term a is 3 and common difference d is -2.



Question 26.

For the following A.P's write the first term and common difference:



Answer:

In general, for an AP a1, a2, . . . .,an, we have


d = ak+1 - ak


where ak+1 and ak are the (k+1)th and kth terms respectively.


For the list of numbers:





Here, the difference of any two consecutive terms in each case is . So, the given list is an AP whose first term a is 2 and common difference d is .



Question 27.

For the following A.P's write the first term and common difference:



Answer:

In general, for an AP a1, a2, . . . .,an, we have


d = ak+1 - ak


where ak+1 and ak are the (k+1)th and kth terms respectively.


For the list of numbers:





Here, the difference of any two consecutive terms in each case is -1. So, the given list is an AP whose first term a is and common difference d is -1.



Question 28.

Which of the following list of numbers form A.R's? If they form an A.P., find the common difference d and also write its next three terms.

1, - 1, - 3, - 5,


Answer:

We have,


a2 – a1 = -1 – 1 = -2


a3 – a2 =-3 – (-1) = -3 + 1 = -2


a4 – a3 =-5 – (-3) = -5 + 3= -2


i.e. ak+1 – ak is the same every time.


So, the given list of numbers forms an AP with the common difference d = -2


Now, we have to find the next three terms.


We have a1 = 1, a2 = -1, a3 = -3 and a4 = -5


Now, we will find a5, a6 and a7


So, a5 = -5 + (-2) = -5 – 2 = -7


a6 = -7 + (-2) = -7 – 2 = -9


and a7 = -9 + (-2) = -9 – 2 = -11


Hence, the next three terms are -7, -9 and -11



Question 29.

Which of the following list of numbers form A.R's? If they form an A.P., find the common difference d and also write its next three terms.

2, 4, 8, 16, ...


Answer:

We have,


a2 – a1 = 4 – 2 = 2


a3 – a2 = 8 – 4 = 4


a4 – a3 = 16 – 8 = 8


i.e. ak+1 – ak is not same every time.


So, the given list of numbers do not form an AP.



Question 30.

Which of the following list of numbers form A.R's? If they form an A.P., find the common difference d and also write its next three terms.

- 2, 2, - 2, 2, - 2, ...


Answer:

We have,


a2 – a1 = 2 – (-2) = 2 + 2 = 4


a3 – a2 = -2 – 2 = -4


a4 – a3 = 2 – (-2) = 2 + 2 = 4


i.e. ak+1 – ak is not same every time.


So, the given list of numbers do not form an AP.



Question 31.

Which of the following list of numbers form A.R's? If they form an A.P., find the common difference d and also write its next three terms.



Answer:

We have,





i.e. ak+1 – ak is the same every time.


So, the given list of numbers forms an AP with the common difference d = 0


Now, we have to find the next three terms.


We have


Now, we will find a5, a6 and a7


So,



and


Hence, the next three terms are



Question 32.

Which of the following list of numbers form A.R's? If they form an A.P., find the common difference d and also write its next three terms.



Answer:

We have,





i.e. ak+1 – ak is the same every time.


So, the given list of numbers forms an AP with the common difference


Now, we have to find the next three terms.


We have


Now, we will find a5, a6 and a7


So,



and


Hence, the next three terms are



Question 33.

Which of the following list of numbers form A.R's? If they form an A.P., find the common difference d and also write its next three terms.

0, - 4, - 8, - 12,


Answer:

We have,


a2 – a1 = -4 – 0 = -4


a3 – a2 =-8 – (-4) = -8 + 4 = -4


a4 – a3 =-12 – (-8) = -12 + 8= -4


i.e. ak+1 – ak is the same every time.


So, the given list of numbers forms an AP with the common difference d = -4


Now, we have to find the next three terms.


We have a1 = 0, a2 = -4, a3 = -8 and a4 = -12


Now, we will find a5, a6 and a7


So, a5 = -12 + (-4) = -12 – 4 = -16


a6 = -16 + (-4) = -16 – 4 = -20


and a7 = -20 + (-4) = -20 – 4 = -24


Hence, the next three terms are -16, -20 and -24



Question 34.

Which of the following list of numbers form A.R's? If they form an A.P., find the common difference d and also write its next three terms.

4, 10, 16, 22, ...


Answer:

We have,


a2 – a1 = 10 – 4 = 6


a3 – a2 = 16 – 10 = 6


a4 – a3 = 22 – 16 = 6


i.e. ak+1 – ak is the same every time.


So, the given list of numbers forms an AP with the common difference d = 6


Now, we have to find the next three terms.


We have a1 = 4, a2 = 10, a3 = 16 and a4 = 22


Now, we will find a5, a6 and a7


So, a5 = 22 + 6 = 28


a6 = 28 + 6 = 34


and a7 = 34 + 6 = 40


Hence, the next three terms are 28, 34 and 40



Question 35.

Which of the following list of numbers form A.R's? If they form an A.P., find the common difference d and also write its next three terms.

a, 2a, 3a, 4a, ...


Answer:

We have,


a2 – a1 = 2a – a = a


a3 – a2 = 3a – 2a = a


a4 – a3 = 4a – 3a = a


i.e. ak+1 – ak is the same every time.


So, the given list of numbers forms an AP with the common difference d = a


Now, we have to find the next three terms.


We have a1 = a, a2 = 2a, a3 = 3a and a4 = 4a


Now, we will find a5, a6 and a7


So, a5 = 4a + a = 5a


a6 = 5a + a = 6a


and a7 = 6a + a = 7a


Hence, the next three terms are 5a, 6a and 7a



Question 36.

Which of the following list of numbers form A.R's? If they form an A.P., find the common difference d and also write its next three terms.

- 1.2, - 3.2, - 5.2, - 7.2, ...


Answer:

We have,


a2 – a1 = -3.2 – (-1.2) =-3.2 + 1.2 = -2.0


a3 – a2 = -5.2 – (-3.2) = -5.2 + 3.2 = -2.0


a4 – a3 = -7.2 – (-5.2) = -7.2 + 5.2 = -2.0


i.e. ak+1 – ak is the same every time.


So, the given list of numbers forms an AP with the common difference d = -2


Now, we have to find the next three terms.


We have a1 = -1.2, a2 = -3.2, a3 = -5.2 and a4 = -7.2


Now, we will find a5, a6 and a7


So, a5 = -7.2 + (-2) = -7.2 – 2.0 = -9.2


a6 = -9.2 + (-2) = -9.2 – 2.0 = -11.2


and a7 = -11.2 + (-2) = -11.2 – 2.0 = -13.2


Hence, the next three terms are -9.2, -11.2 and -13.2



Question 37.

Which of the following list of numbers form A.R's? If they form an A.P., find the common difference d and also write its next three terms.



Answer:

We have,


a2 – a1 = √12 - √3 = 2√3 - √3 = √3


a3 – a2 = √48 - √12 = 4√3 - 2√3 = 2√3


a4 – a3 = √192 - √48 = 8√3 - 4√3 = 4√3


i.e. ak+1 – ak is not same every time.


So, the given list of numbers do not form an AP.



Question 38.

Which of the following list of numbers form A.R's? If they form an A.P., find the common difference d and also write its next three terms.

a, a2, a3, a4, …..


Answer:

We have,


a2 – a1 = a2 – a = a (a – 1)


a3 – a2 = a3 – a2 = a2(a – 1)


a4 – a3 = a4 – a3 = a3(a – 1)


i.e. ak+1 – ak is not same every time.


So, the given list of numbers does not form an AP.



Question 39.

Which of the following list of numbers form A.R's? If they form an A.P., find the common difference d and also write its next three terms.

1, 3, 9, 27, ...


Answer:

We have,


a2 – a1 = 3 – 1 = 2


a3 – a2 = 9 – 3 = 6


a4 – a3 = 27 – 9 = 18


i.e. ak+1 – ak is not same every time.


So, the given list of numbers does not form an AP.



Question 40.

Which of the following list of numbers form A.R's? If they form an A.P., find the common difference d and also write its next three terms.

12, 22, 32, 42, …


Answer:

We have,


a2 – a1 = 22 – (1)2 = 4 – 1 = 3


a3 – a2 = 32 – (2)2 = 9 – 4 = 5


a4 – a3 = 42 – (3)2 = 16 – 9 =7


i.e. ak+1 – ak is not same every time.


So, the given list of numbers does not form an AP.



Question 41.

Which of the following list of numbers form A.R's? If they form an A.P., find the common difference d and also write its next three terms.

12, 52, 72, 72, …


Answer:

We have,


a2 – a1 = 52 – (1)2 = 25 – 1 = 24


a3 – a2 = 72 – (5)2 = 49 – 25 = 24


a4 – a3 = 72 – (7)2 = 0


i.e. ak+1 – ak is not same every time.


So, the given list of numbers do not form an AP.



Question 42.

Which of the following list of numbers form A.R's? If they form an A.P., find the common difference d and also write its next three terms.

12, 32, 52, 72, …


Answer:

We have,


a2 – a1 = 32 – (1)2 = 9 – 1 = 8


a3 – a2 = 52 – (3)2 = 25 – 9 = 16


a4 – a3 = 72 – (5)2 = 49 – 25 =24


i.e. ak+1 – ak is not same every time.


So, the given list of numbers does not form an AP.



Question 43.

In which of the following situations does the list of numbers involved arithmetic progression, and why?

The salary of a teacher in successive years when starting salary is Rs. 8000, with an annual increment of Rs. 500.


Answer:

Salary for the 1st year = Rs. 8000


and according to the question,


There is an annual increment of Rs. 500


⇒ The salary for the 2nd year = Rs. 8000 + 500 = Rs.8500


Now, again there is an increment of Rs. 500


⇒ The salary for the 3rd year = Rs. 8500 +500 = Rs. 9000


Therefore, the series is


8000 , 8500 , 9000 , …


Difference between 2nd term and 1st term = 8500 – 8000 = 500


Difference between 3rd term and 2nd term = 9000 – 8500 = 500


Since, the difference is same.


Hence, salary in successive years are in AP with common difference d = 500 and first term a is 8000.



Question 44.

In which of the following situations does the list of numbers involved arithmetic progression, and why?

The taxi fare after each km when the fare is Rs. 15 for the first km and Rs. 8 for each additional km.


Answer:

Taxi fare for 1km = 15


According to question, Rs. 8 for each additional km


⇒ Taxi fare for 2km = 15 + 8 =23


and Taxi fare for 3km = 23 + 8 =31


Therefore, series is


15, 23, 31 ,…


Difference between 2nd and 1st term = 23 – 15 = 8


Difference between 3rd and 2nd term = 31 – 23 = 8


Since, difference is same.


Hence, the taxi fare after each km form an AP with the first term, a = Rs. 15 and common difference, d = Rs. 8



Question 45.

In which of the following situations does the list of numbers involved arithmetic progression, and why?

The lengths of the rungs of a ladder when the bottom rung is 45cm, and length of rungs decrease by 2 cm from bottom to top.


Answer:


The length of the bottom rung = 45cm


According to the question,


Length of rungs decreases by 2cm from bottom to top. The lengths (in cm) of the 1st, 2nd, 3rd, … from the bottom to top respectively are 45, 43, 41, …


Difference between 2nd and 1st term = 43 – 45 = -2


Difference between 3rd and 2nd term = 41 – 43 = -2


Since, the difference is same.


Hence, the length of the rungs form an AP with a = 45cm and d = -2 cm.



Question 46.

In which of the following situations does the list of numbers involved arithmetic progression, and why?

The amount of money in the account every year when Rs. 10000 is deposited at compound interest 8% per annum.


Answer:

Original Amount = Rs. 10,000


Interest earned in first year = 10,000 × 8%



= Rs 800


Total amount outstanding after one year = Rs 10000 + 800


= Rs 10800


Now, interest earned in 2nd year


Total amount outstanding after 2nd year = Rs10800 + 864


= Rs 11664


Interest earned in 3rd year


= Rs 933.12


Total amount outstanding after 3rd year = Rs 11664 + 933.12


= Rs 12597.12


Therefore, the series is


10800, 11664, 12597.12,…


Difference between second and first term = 11664 – 10800


= 864


Difference between third and second term = 12597.12 – 11664


= 933.12


Since the difference is not same


Therefore, it doesn’t form an AP.



Question 47.

In which of the following situations does the list of numbers involved arithmetic progression, and why?

The money saved by Sudha in successive years when she saves Rs. 100 the first year and increased the amount by Rs. 50 every year.


Answer:

The money saved by Sudha in the first year = Rs. 100


According to the question,


Sudha increased the amount by Rs. 50 every year


⇒ The money saved by Sudha in a 2nd year = Rs. 100 +50


= Rs. 150


The money saved by Sudha in a 3rd year = Rs. 150 + 50


= Rs. 200


Therefore, the series is


100, 150, 200, 250,…


Difference in the 2nd term and 1st term = 150 – 100 = 50


Difference in the 3rd term and 2nd term = 200 – 150 = 50


Since the difference is the same.


Therefore, the money saved by Sudha in successive years form an AP with a = Rs 100 and d =Rs 50



Question 48.

In which of the following situations does the list of numbers involved arithmetic progression, and why?

Number of pairs of rabbits in successive months when the pair of rabbits is too young to produce in their first month. In the second month and every subsequent month, they produce a new pair. Each new pair of rabbits pr a new pair in their second months and every subsequent month (see Fig.) (assume that no rabbit dies).



Answer:

Assuming that no rabbit dies,


the number of pairs of rabbits at the start of the 1st month = 1


the number of pairs of rabbits at the start of the 2nd month = 1


the number of pairs of rabbits at the start of the 3rd month = 2


the number of pairs of rabbits at the start of the 4th month = 3


the number of pairs of rabbits at the start of the 5th month = 5


Therefore, the series is


1, 1, 2, 3, 5, 8,…


Difference between 2nd and 1st term = 1 – 1 = 0


Difference between 3rd and 2nd term = 2 – 1 = 1


Since, the difference is not same.


Therefore, the number of pair of rabbits in successive months are 1,1,2,3,5,8,… and they don’t form an AP.



Question 49.

In which of the following situations does the list of numbers involved arithmetic progression, and why?
The values of an investment after 1, 2, 3, 4, ... years if after each subsequent year it increases by 5/4 times the initial investment.


Answer:

Let the initial investment be I,


After one year it increases by I,


So the investment becomes, I + I


= I


At the end of 2nd year it again increase to I,


So the investment becomes, (I + I + I )


=( I + I)


= I


At the end of the 3rd year it again increases to I


So the investment becomes (I + I + I + I )


= I + I + I


= I+ I


= I


Therefore the series is:


I, I, I, I,………


Now difference between 2nd and 1st term is I – I = I


difference between 3rd and 2nd term is I I = I


Since the difference is same,


Hence the obtained series is an A.P.




Exercise 8.2
Question 1.

Find the indicated terms in each of the following arithmetic progression:

1, 6, 11, 16, ..., t16,


Answer:

Given: 1, 6, 11, 16, …


Here, a = 1


d = a2 – a1 = 6 – 1 = 5


and n = 16


We have,


tn = a + (n – 1)d


So, t16 = 1 + (16 – 1)5


= 1 + 15×5


t16 = 1 +75


t16 = 76



Question 2.

Find the indicated terms in each of the following arithmetic progression:

a = 3, d = 2; ; tn, t10


Answer:

Given: a = 3 , d = 2


To find: tn and t10


We have,


tn = a + (n – 1)d


tn = 3 + (n – 1) 2


= 3 + 2n – 2


tn = 2n + 1


Now, n = 10


So, t10 = 3 + (10 – 1)2


= 3 + 9×2


t10 = 3 +18


t10 = 21



Question 3.

Find the indicated terms in each of the following arithmetic progression:

— 3, — 1/2, 2, ... ; t10,


Answer:

Given:


Here, a = –3



and n = 10


We have,


tn = a + (n – 1)d


So,






Question 4.

Find the indicated terms in each of the following arithmetic progression:

a = 21, d = — 5; tn, t25


Answer:

Given: a = 21 , d = –5


To find: tn and t25


We have,


tn = a + (n – 1)d


tn = 21 + (n – 1)(–5)


= 21 – 5n + 5


tn = 26 – 5n


Now, n = 25


So, t25 = 21 + (25 – 1)(–5)


= 21 + 24 × (–5)


t25 = 21 – 120


t25 = –99



Question 5.

Find the 10th term of the A.P. 10, 5, 0, — 5, — 10, ...


Answer:

Given: 10, 5, 0, –5, –10,…


To find: 10th term i.e. t10


Here, a = 10


d = a2 – a1 = 5 – 10 = –5


and n = 10


We have,


tn = a + (n – 1)d


t10 = 10 + (10 – 1)(–5)


= 10 + 9 × –5


t10 = 10 – 45


t10 = –35


Therefore, the 10th term of the given is –35.



Question 6.

Find the 10th term of the A.P.


Answer:

Given:


Here,



and n = 10


We have,


tn = a + (n – 1)d






Therefore, the 10th term of the given AP is



Question 7.

Find the sum of 20th and 25th terms of A.P. 2, 5, 8, 11, ...


Answer:

Given: 2, 5, 8, 11, …


Here, a = 2


d = a2 – a1 = 5 – 2 = 3


and n = 20


We have,


tn = a + (n – 1)d


t20 = 2 + (20 – 1)(3)


t20 = 2 + 19 × 3


= 2 + 57


t20 = 59


Now, n = 25


t25 = 2 + (25 – 1)(3)


t25 = 2 + 24 × 3


t25 = 2 + 72


t25 = 74


The sum of 20th and 25th terms of AP = t20 + t25 = 59 + 74 = 133



Question 8.

Find the number of terms in the following A.P.'s

6, 3, 0, — 3,…..,–36


Answer:

Here, a = 6 , d = 3 – 6 = –3 and l = –36


where l = a + (n – 1)d


⇒ –36 = 6 + (n – 1)(–3)


⇒ –36 = 6 –3n + 3


⇒ –36 = 9 – 3n


⇒ –36 – 9 = –3n


⇒ –45 = –3n



Hence, the number of terms in the given AP is 15



Question 9.

Find the number of terms in the following A.P.'s



Answer:

Here,



And


We have,


l = a + (n – 1)d





⇒ 15 = n – 1


⇒ n = 16


Hence, the number of terms in the given AP is 16.



Question 10.

Determine the number of terms in the A.P. 3, 7, 11, ..., 399. Also, find its 20th term from the end.


Answer:

Here, a = 3, d = 7 – 3 = 4 and l = 399


To find : n and 20th term from the end


We have,


l = a + (n – 1)d


⇒ 399 = 3 + (n – 1) × 4


⇒ 399 – 3 = 4n – 4


⇒ 396 + 4 = 4n


⇒ 400 = 4n


⇒ n = 100


So, there are 100 terms in the given AP


Last term = 100th


Second Last term = 100 – 1 = 99th


Third last term = 100 – 2 = 98th


And so, on


20th term from the end = 100 – 19 = 81st term


The 20th term from the end will be the 81st term.


So, t81 = 3 + (81 – 1)(4)


t81 = 3 + 80 × 4


t81 = 3 + 320


t81 = 323


Hence, the number of terms in the given AP is 100, and the 20th term from the last is 323.



Question 11.

Which term of the A.P. 5, 9, 13, 17, ... is 81?


Answer:

Here, a = 5, d = 9 – 5 = 4 and an = 81


To find : n


We have,


an = a + (n – 1)d


⇒ 81 = 5 + (n – 1) × 4


⇒ 81 = 5 + 4n – 4


⇒ 81 = 4n + 1


⇒ 80 = 4n


⇒ n = 20


Therefore, the 20th term of the given AP is 81.



Question 12.

Which term of the A.P. 14, 9, 4, – I, – 6, ... is – 41 ?


Answer:

Here, a = 14, d = 9 – 14 = –5 and an = –41


To find : n


We have,


an = a + (n – 1)d


⇒ –41 = 14 + (n – 1) × (–5)


⇒ –41 = 14 – 5n + 5


⇒ –41 = 19 – 5n


⇒ – 41 – 19 = –5n


⇒ –60 = –5n


⇒ n = 12


Therefore, the 12th term of the given AP is –41.



Question 13.

Which term of A.P. 3, 8, 13, 18, ... is 88?


Answer:

Here, a = 3, d = 8 – 3 = 5 and an = 88


To find : n


We have,


an = a + (n – 1)d


⇒ 88 = 3 + (n – 1) × (5)


⇒ 88 = 3 + 5n – 5


⇒ 88 = –2 + 5n


⇒88 + 2 = 5n


⇒ 90 = 5n


⇒ n = 18


Therefore, the 18th term of the given AP is 88.



Question 14.

Which term of A.P. is 3?


Answer:

Here,



and an = 3


We have,


an = a + (n – 1)d





⇒ 13 = n – 1


⇒ n = 14


Therefore, the 14th term of a given AP is 3.



Question 15.

Which term of A.P. 3, 8, 13, 18, ..., is 248 ?


Answer:

Here, a = 3, d = 8 – 3 = 5 and an = 248


To find : n


We have,


an = a + (n – 1)d


⇒ 248 = 3 + (n – 1) × (5)


⇒ 248 = 3 + 5n – 5


⇒ 248 = –2 + 5n


⇒ 248 + 2 = 5n


⇒ 250 = 5n


⇒ n = 50


Therefore, the 50th term of the given AP is 248.



Question 16.

Find the 6th term from end of the A.P. 17, 14, 11,… – 40.


Answer:

Here, a = 17, d = 14 – 17 = –3 and l = –40


where l = a + (n – 1)d


Now, to find the 6th term from the end, we will find the total number of terms in the AP.


So, –40 = 17 + (n – 1)(–3)


⇒ –40 = 17 –3n + 3


⇒ –40 = 20 – 3n


⇒ –60 = –3n


⇒ n = 20


So, there are 20 terms in the given AP.


Last term = 20th


Second last term = 20 – 1 = 19th


Third last term = 20 – 2 = 18th


And so, on


So, the 6th term from the end = 20 – 5 = 15th term


So, an = a + (n – 1)d


⇒ a15 = 17 + (15 – 1)(–3)


⇒ a15 = 17 + 14 × –3


⇒ a15 = 17 – 42


⇒ a15 = –25



Question 17.

Find the 8th term from end of the A.P. 7, 10, 13, ..., 184.


Answer:

Here, a = 7, d = 10 – 7 = 3 and l = 184


where l = a + (n – 1)d


Now, to find the 8th term from the end, we will find the total number of terms in the AP.


So, 184 = 7 + (n – 1)(3)


⇒ 184 = 7 + 3n – 3


⇒ 184 = 4 + 3n


⇒ 180 = 3n


⇒ n = 60


So, there are 60 terms in the given AP.


Last term = 60th


Second last term = 60 – 1 = 59th


Third last term = 60 – 2 = 58th


And so, on


So, the 8th term from the end = 60 – 7 = 53th term


So, an = a + (n – 1)d


⇒ a53 = 7 + (53 – 1)(3)


⇒ a53 = 7 + 52 × 3


⇒ a53 = 7 + 156


⇒ a53 = 163



Question 18.

Find the number of terms of the A.P.

6, 10, 14, 18, ..., 174?


Answer:

Here, a = 6 , d = 10 – 6 = 4 and l = 174


where l = a + (n – 1)d


⇒ 174 = 6 + (n – 1)(4)


⇒ 174 = 6 + 4n – 4


⇒ 174 = 2 + 4n


⇒ 174 – 2 = 4n


⇒ 172 = 4n



Hence, the number of terms in the given AP is 43



Question 19.

Find the number of terms of the A.P.

7, 11, 15, ..., 139?


Answer:

Here, a = 7 , d = 11 – 7 = 4 and l = 139


where l = a + (n – 1)d


⇒ 139 = 7 + (n – 1)(4)


⇒ 139 = 7 + 4n – 4


⇒ 139 = 3 + 4n


⇒ 139 – 3 = 4n


⇒ 136 = 4n



Hence, the number of terms in the given AP is 34



Question 20.

Find the number of terms of the A.P.

41, 38, 35, ..., 8?


Answer:

Here, a = 41 , d = 38 – 41 = –3 and l = 8


where l = a + (n – 1)d


⇒ 8 = 41 + (n – 1)(–3)


⇒ 8 = 41 –3n + 3


⇒ 8 = 44 – 3n


⇒ 8 – 44 = –3n


⇒ –36 = –3n



Hence, the number of terms in the given AP is 12



Question 21.

Find the first negative term of sequence 999, 995, 991, 987, ...


Answer:

AP = 999, 995, 991, 987,…


Here, a = 999, d = 995 – 999 = –4


an < 0


⇒ a + (n – 1)d < 0


⇒ 999 + (n – 1)(–4) < 0


⇒ 999 – 4n + 4 < 0


⇒ 1003 – 4n < 0


⇒ 1003 < 4n



⇒ n > 250.75


Nearest term greater than 250.75 is 251


So, 251st term is the first negative term


Now, we will find the 251st term


an = a +(n – 1)d


⇒ a251 = 999 + (251 – 1)(–4)


⇒ a251 = 999 + 250 × –4


⇒ a251 = 999 – 1000


⇒ a251 = – 1


∴, –1 is the first negative term of the given AP.



Question 22.

Is 51 a term of the A.P. 5, 8, 11, 14, ...?


Answer:

AP = 5, 8, 11, 14, …


Here, a = 5 and d = 8 – 5 = 3


Let 51 be a term, say, nth term of this AP.


We know that


an = a + (n – 1)d


So, 51 = 5 + (n – 1)(3)


⇒ 51 = 5 + 3n – 3


⇒ 51 = 2 + 3n


⇒ 51 – 2 = 3n


⇒ 49 = 3n



But n should be a positive integer because n is the number of terms. So, 51 is not a term of this given AP.



Question 23.

Is 56 a term of the A.P.


Answer:

AP


Here, a = 4 and


Let 56 be a term, say, nth term of this AP.


We know that


an = a + (n – 1)d


So,


⇒ 2 × (56 – 4) = n – 1


⇒ 2 × 52 = n – 1


⇒ 104 = n – 1


⇒ 105 = n


Hence, 56 is the 105th term of this given AP.



Question 24.

The 7th term of an A.P. is 20 and its 13th term is 32. Find the A.P. [CBSE 20041


Answer:

We have


a7 = a + (7 – 1)d = a + 6d = 20 …(1)


and a13 = a + (13 – 1)d = a + 12d = 32 …(2)


Solving the pair of linear equations (1) and (2), we get


a + 6d – a – 12d = 20 – 32


⇒ – 6d = –12


⇒ d = 2


Putting the value of d in eq (1), we get


a + 6(2) = 20


⇒ a + 12 = 20


⇒ a = 8


Hence, the required AP is 8, 10, 12, 14,…



Question 25.

The 7th term of an A.P. is – 4 and its 13th term is – 16. Find the A.P. [CBSE 20041


Answer:

We have


a7 = a + (7 – 1)d = a + 6d = –4 …(1)


and a13 = a + (13 – 1)d = a + 12d = –16 …(2)


Solving the pair of linear equations (1) and (2), we get


a + 6d – a – 12d = –4 – (–16)


⇒ – 6d = –4 + 16


⇒ – 6d = 12


⇒ d = –2


Putting the value of d in eq (1), we get


a + 6(–2) = –4


⇒ a – 12 = –4


⇒ a = 8


Hence, the required AP is 8, 6, 4, 2,…



Question 26.

The 8th term of an A.P. is 37, and its 12th term is 57. Find the A.P.


Answer:

We have


a8 = a + (8 – 1)d = a + 7d = 37 …(1)


and a12 = a + (12 – 1)d = a + 11d = 57 …(2)


Solving the pair of linear equations (1) and (2), we get


a + 7d – a – 11d = 37 – 57


⇒ – 4d = –20


⇒ d = 5


Putting the value of d in eq (1), we get


a + 7(5) = 37


⇒ a + 35 = 37


⇒ a = 2


Hence, the required AP is 2, 7, 12, 17,…



Question 27.

Find the 10th term of the A.P. whose 7th and 12th terms are 34 and 64 respectively.


Answer:

We have


a7 = a + (7 – 1)d = a + 6d = 34 …(1)


and a12 = a + (12 – 1)d = a + 11d = 64 …(2)


Solving the pair of linear equations (1) and (2), we get


a + 6d – a – 11d = 34 – 64


⇒ – 5d = –30


⇒ d = 6


Putting the value of d in eq (1), we get


a + 6(6) = 34


⇒ a + 36 = 34


⇒ a = –2


Hence, the required AP is –2, 4, 10, 16,…


Now, we to find the 10th term


So, an = a + (n – 1)d


a10 = –2 + (10 – 1)6


a10 = –2 + 9 × 6


a10 = 52



Question 28.

For what value of n are the nth term of the following two A.P's the same. Also find this term

13, 19, 25, ... and 69, 68, 67, ...


Answer:

1st AP = 13, 19, 25, …


Here, a = 13, d = 19 – 13 = 6


and 2nd AP = 69, 68, 67, …


Here, a = 69, d = 68 – 69 = –1


According to the question,


13 + (n – 1)6 = 69 + (n – 1)(–1)


⇒ 13 + 6n – 6 = 69 – n + 1


⇒ 7 + 6n = 70 – n


⇒ 6n + n = 70 – 7


⇒ 7n = 63


⇒ n = 9


9th term of the given AP’s are same.


Now, we will find the 9th term


We have,


an = a + (n – 1)d


a9 = 13 + (9 – 1)6


a9 = 13 + 8 × 6


a9 = 13 + 48


a9 = 61



Question 29.

For what value of n are the nth term of the following two A.P's the same. Also find this term

23, 25, 27, 29, ... and – 17, – 10, – 3, 4, ...


Answer:

1st AP = 23, 25, 27, 29, ...


Here, a = 23, d = 25 – 23 = 2


and 2nd AP = – 17, – 10, – 3, 4, ...


Here, a = –17, d = –10 – (–17) = –10 + 17 = 7


According to the question,


23 + (n – 1)2 = –17 + (n – 1)7


⇒ 23 + 2n – 2 = –17 + 7n – 7


⇒ 21 + 2n = –24 + 7n


⇒ 2n – 7n = –24 – 21


⇒ –5n = –45


⇒ n = 9


9th term of the given AP’s are same.


Now, we will find the 9th term


We have,


an = a + (n – 1)d


a9 = 23 + (9 – 1)2


a9 = 23 + 8 × 2


a9 = 23 + 16


a9 = 39



Question 30.

For what value of n are the nth term of the following two A.P's the same. Also find this term

24, 20, 16, 12, ... and – 11, – 8, – 5, – 2, ...


Answer:

1st AP = 24, 20, 16, 12, ...


Here, a = 24, d = 20 – 24 = –4


and 2nd AP = – 11, – 8, – 5, – 2, ...


Here, a = –11, d = –8 – (–11) = –8 + 11 = 3


According to the question,


24 + (n – 1)(–4) = –11 + (n – 1)3


⇒ 24 – 4n + 4 = –11 + 3n – 3


⇒ 28 – 4n = –14 + 3n


⇒ 28 + 14 = 3n + 4n


⇒ 7n = 42


⇒ n = 6


6th term of the given AP’s are same.


Now, we will find the 6th term


We have,


an = a + (n – 1)d


a6 = 24 + (6 – 1)(–4)


a6 = 24 + 5 × –4


a6 = 24 – 20


a6 = 4



Question 31.

For what value of n are the nth term of the following two A.P's the same. Also find this term

63, 65, 67, ... and 3, 10, 17, ...


Answer:

1st AP = 63, 65, 67, ...


Here, a = 63, d = 65 – 63 = 2


and 2nd AP = 3, 10, 17, ...


Here, a = 3, d = 10 – 3 = 7


According to the question,


63 + (n – 1)2 = 3 + (n – 1)7


⇒ 63 + 2n – 2 = 3 + 7n – 7


⇒ 61 + 2n = 7n – 4


⇒ 65 = 7n – 2n


⇒ 5n = 65


⇒ n = 13


13th term of the given AP’s are same.


Now, we will find the 13th term


We have,


an = a + (n – 1)d


a13 = 63 + (13 – 1)2


a13 = 63 + 12 × 2


a13 = 63 + 24


a13 = 87



Question 32.

In the following A.P., find the missing terms:

5, □,□, 9


Answer:

Here, a = 5 , n = 4 and


We have,


l = a + (n – 1)d



⇒19 = 10 + 6d


⇒9 = 6d



So, the missing terms are –


a2 = a + d


a3 = a + 2d


Hence, the missing terms are



Question 33.

In the following A.P., find the missing terms:

54, □,□, 42


Answer:

Here, a = 54 , n = 4 and l = 42


We have,


l = a + (n – 1)d


⇒42 = 54 + (4 – 1)d


⇒42 = 54 + 3d


⇒–12 = 3d



So, the missing terms are –


a2 = a + d = 54 – 4 = 50


a3 = a + 2d = 54 + 2(–4) = 54 – 8 = 46


Hence, the missing terms are 50 and 46



Question 34.

In the following A.P., find the missing terms:

– 4, □,□,□,□, 6


Answer:

Here, a = –4, n = 6 and l = 6


We have,


l = a + (n – 1)d


⇒6 = –4 + (6 – 1)d


⇒6 = –4 + 5d


⇒10 = 5d



So, the missing terms are –


a2 = a + d = –4 + 2 = –2


a3 = a + 2d = –4 + 2(2) = –4 + 4 = 0


a4 = a + 3d = –4 + 3(2) = –4 + 6 = 2


a5 = a + 4d = –4 + 4(2) = –4 + 8 = 4


Hence, the missing terms are –2, 0, 2 and 4



Question 35.

In the following A.P., find the missing terms:

□, 13, □, 3


Answer:

Given: a2 = 13 and a4 = 3


We know that,


an = a + (n – 1)d


a2 = a + (2 – 1)d


13 = a + d …(i)


and a4 = a +(4 – 1)d


3 = a + 3d …(ii)


Solving linear equations (i) and (ii), we get


a + d – a – 3d = 13 – 3


⇒ –2d = 10


⇒ d = –5


Putting the value of d in eq. (i), we get


a – 5 = 13


⇒ a = 18


Now, a3 = a + 2d = 18 + 2(–5) = 18 – 10 = 8


Hence, the missing terms are 18 and 8



Question 36.

In the following A.P., find the missing terms:

7, □,□,□,27


Answer:

Here, a = 7, n = 5 and l = 27


We have,


l = a + (n – 1)d


⇒27 = 7 + (5 – 1)d


⇒27 = 7 + 4d


⇒20 = 4d



So, the missing terms are –


a2 = a + d = 7 + 5 = 12


a3 = a + 2d = 7 + 2(5) = 7 + 10 = 17


a4 = a + 3d = 7 + 3(5) = 7 + 15 = 22


Hence, the missing terms are 12, 17 and 22



Question 37.

In the following A.P., find the missing terms:

2, □, 26


Answer:

Here, a = 2, n = 3 and l = 26


We have,


l = a + (n – 1)d


⇒26 = 2 + (3 – 1)d


⇒26 = 2 + 2d


⇒24 = 2d



So, the missing terms are –


a2 = a + d = 2 + 12 = 14


Hence, the missing terms is 14



Question 38.

In the following A.P., find the missing terms:

□, □, 13, □, □, 22


Answer:

Given: a3 = 13 and a6 = 22


We know that,


an = a + (n – 1)d


a3 = a + (3 – 1)d


13 = a + 2d …(i)


and a6 = a +(6 – 1)d


22 = a + 5d …(ii)


Solving linear equations (i) and (ii), we get


a + 2d – a – 5d = 13 – 22


⇒ –3d = 9


⇒ d = 3


Putting the value of d in eq. (i), we get


a + 2(3) = 13


⇒ a + 6 = 13


⇒ a = 7


Now, a2 = a + d = 7 + 3 = 10


a4 = a + 3d = 7 + 3(3) = 7 + 9 = 16


a5 = a + 4d = 7 + 4(3) = 7 + 12 = 19


Hence, the missing terms are 7, 10, 16 and 19



Question 39.

In the following A.P., find the missing terms:

– 4, □, □, □, 6


Answer:

Here, a = –4, n = 5 and l = 6


We have,


l = a + (n – 1)d


⇒6 = –4 + (5 – 1)d


⇒6 = –4 + 4d


⇒10 = 4d



So, the missing terms are –


a2 = a + d


a3 = a + 2d


a4 = a + 3d =


Hence, the missing terms are



Question 40.

In the following A.P., find the missing terms:

□, 38, □,□,□, – 22


Answer:

Given: a2 = 38 and a6 = –22


We know that,


an = a + (n – 1)d


a2 = a + (2 – 1)d


38 = a + d …(i)


and a6 = a +(6 – 1)d


–22 = a + 5d …(ii)


Solving linear equations (i) and (ii), we get


a + d – a – 5d = 38 – (–22)


⇒ –4d = 60


⇒ d = –15


Putting the value of d in eq. (i), we get


a + (–15) = 38


⇒ a – 15 = 38


⇒ a = 53


Now, a3 = a + 2d = 53 + 2(–15) = 53 – 30 = 23


a4 = a + 3d = 53 + 3(–15) = 53 – 45 = 8


a5 = a + 4d = 53 + 4(–15) = 53 – 60 = –7


Hence, the missing terms are 53, 23, 8 and –7



Question 41.

If 10th term of an A.P. is 52 and 17th term is 20 more than the 13th term, find the A.P.


Answer:

Given: a10 = 52 and a17 = 20 + a13


Now, an = a + (n – 1)d


a10 = a + (10 – 1)d


52 = a + 9d …(i)


and a17 = 20 + a13


a + (17 – 1)d = 20 + a + (13 – 1)d


⇒ a + 16d = 20 + a + 12d


⇒ 16d –12d = 20


⇒ 4d = 20


⇒ d = 5


Putting the value of d in eq. (i), we get


a + 9(5) = 52


⇒ a + 45 = 52


⇒ a = 52 – 45


⇒ a = 7


Therefore, the AP is 7, 12, 17, …



Question 42.

Which term of the A.P. 3, 15, 27, 39, ... will be 132 more than its 54th term?


Answer:

Given: 3, 15, 27, 39, …


First we need to calculate 54th term.


We know that


an = a + (n – 1)d


Here, a = 3, d = 15 – 3 = 12 and n = 54


So, a54 = 3 + (54 – 1)12


⇒ a54 = 3 + 53 × 12


⇒ a54 = 3 + 636


⇒ a54 = 639


Now, the term is 132 more than a54 is 132 + 639 = 771


Now,


a + (n – 1)d = 771


⇒ 3 + (n – 1)12 = 771


⇒ 3 + 12n – 12 = 771


⇒ 12n = 771 + 12 – 3


⇒ 12n = 780


⇒ n = 65


Hence, the 65th term is 132 more than the 54th term.



Question 43.

Which term of the A.P. 3, 10, 17, 24, ... will be 84 more than its 13th term ?


Answer:

Given: 3, 10, 17, 24, ...


First we need to calculate 13th term.


We know that


an = a + (n – 1)d


Here, a = 3, d = 10 – 3 = 7 and n = 13


So, a13 = 3 + (13 – 1)7


⇒ a13 = 3 + 12 × 7


⇒ a13 = 3 + 84


⇒ a13 = 87


Now, the term is 84 more than a13 is 84 + 87 = 171


Now,


a + (n – 1)d = 171


⇒ 3 + (n – 1)7 = 171


⇒ 3 + 7n – 7 = 171


⇒ 7n = 171 + 7 – 3


⇒ 7n = 175


⇒ n = 25


Hence, the 25th term is 84 more than the 13th term.



Question 44.

The 4th term of an A.P. is zero. Prove that its 25th term is triple its 11th term.


Answer:

Given: a4 = 0


To Prove: a25 = 3 × a11


Now, a4 = 0


⇒ a + 3d = 0


⇒ a = –3d


We know that,


an = a + (n – 1)d


a11 = –3d + (11 – 1)d [from (i)]


a11 = –3d + 10d


a11 = 7d …(ii)


Now,


a25 = a + (25 – 1)d


a25 = –3d + 24d [from(i)]


a25 = 21d


a25 = 3 × 7d


a25 = 3 × a11 [from(ii)]


Hence Proved



Question 45.

If 10 times the 10th term of an A.P. is equal to 15 times the 15th term, show that its 25th term is zero.


Answer:

Given: 10 × a10 = 15 × a15


To Prove: a25 = 0


Now,


10 × (a + 9d) = 15 × (a + 14d)


⇒ 10a + 90d = 15a + 210d


⇒ 10a – 15a = 210d – 90d


⇒ –5a = 120d


⇒ a = –24d …(i)


Now,


an = a + (n – 1)d


a25 = –24d + (25 – 1)d [from (i)]


a25 = –24d + 24d


a25 = 0


Hence Proved



Question 46.

If (m + 1)th term of an A.P. is twice the (n + 1)th term, prove that (3m + 1)th term is twice the (m + n + 1)th term.


Answer:

Given: am+1 = 2an+1


To Prove: a3m+1 = 2am+n+1


Now,


an = a + (n – 1)d


⇒ am+1 = a + (m + 1 – 1)d


⇒ am+1 = a + md


and an+1 = a + (n + 1 – 1)d


⇒ an+1 = a + nd


Given: am+1 = 2an+1


a +md = 2(a + nd)


⇒ a + md = 2a + 2nd


⇒ md – 2nd = 2a – a


⇒ d(m – 2n) = a …(i)


Now,


am+n+1 = a + (m + n + 1 – 1)d


= a + (m + n)d


= md – 2nd + md + nd [from (i)]


= 2md – nd


am+n+1 = d (2m – n) …(ii)


a3m+1 = a + (3m + 1 – 1)d


= a + 3md


= md – 2nd + 3md [from (i)]


= 4md – 2nd


= 2d( 2m – n)


a3m+1 = 2am+n+1 [from (ii)]


Hence Proved



Question 47.

If tn be the nth term of an A.P. such that find .


Answer:

Given:


To find:


We know that,


tn = a + (n – 1)d


So,




⇒ 3(a+3d) = 2(a+6d)


⇒ 3a + 9d = 2a + 12d


⇒ 3a – 2a = 12d – 9d


⇒ a = 3d …(i)


Now, [from (i)]



Question 48.

Find the number of all positive integers of 3 digits which are divisible by 5.


Answer:

The list of 3 digit numbers divisible by 5 is:


100, 105, 110,…,995


Here a = 100, d = 105 – 100 = 5, an = 995


We know that


an = a + (n – 1)d


995 = 100 + (n – 1)5


⇒ 895 = (n – 1)5


⇒ 179 = n – 1


⇒ 180 = n


So, there are 180 three– digit numbers divisible by 5.



Question 49.

How many three digit numbers are divisible by 7.


Answer:

The list of 3 digit numbers divisible by 7 is:


105, 112, 119,…,994


Here a = 105, d = 112 – 105 = 7, an = 994


We know that


an = a + (n – 1)d


994 = 105 + (n – 1)7


⇒ 889 = (n – 1)7


⇒ 127 = n – 1


⇒ 128 = n


So, there are 128 three– digit numbers divisible by 7.



Question 50.

If tn denotes the nth term of an A.P., show that tm + t2n+m = 2 tm+n.


Answer:

To show: tm + t2n+m = 2 tm+n


Taking LHS


tm + t2n+m = a + (m – 1)d + a + (2n + m – 1)d


= 2a + md – d + 2nd + md – d


= 2a + 2md + 2nd – 2d


= 2 {a + (m + n – 1)d}


= 2tm+n


= RHS


∴LHS = RHS


Hence Proved



Question 51.

Find a if 5a + 2, 4a – I, a + 2 are in A.P.


Answer:

Let 5a + 2, 4a – 1, a + 2 are in AP


So, first term a = 5a + 2


d = 4a – 1 – 5a – 2 = – a – 3


n = 3


l = a + 2


So,


l = a + (n – 1)d


⇒ a + 2 =5a + 2 + (3 – 1)(–a – 3)


⇒ a + 2 – 5a – 2 = –3a – 9 + a + 3


⇒ – 4a = –2a – 6


⇒ – 4a + 2a = – 6


⇒ –2a = – 6


⇒ a = 3



Question 52.

nth term of a sequence is 2n + 1. Is this sequence an A.P.? If so find its first term and common difference.


Answer:

We know that nth term of an A.P is given by,


an = a + (n – 1) d


Now equating it with the expression given we get,


2 n + 1 = a + (n – 1) d


2 n + 1 = a + nd – d


2 n + 1 = nd + (a – d)


Equating both sides we get,


d = 2 and a – d = 1


So we get,


a = 3 and d = 2.


So the first term of this sequence is 3, and the common difference is 2.



Question 53.

The sum of the 4th and Sth terms of an A.P. is 24 and the sum of the 6th and 10th terms is 44. Find the first three terms of A.P.


Answer:

Given: a4 + a8 = 24


⇒ a +3d + a + 7d = 24


⇒ 2a + 10d = 24 …(i)


and a6 + a10 = 44


⇒ a +5d + a + 9d = 44


⇒ 2a + 14d = 44 …(ii)


Solving Linear equations (i) and (ii), we get


2a + 10d – 2a – 14d = 24 – 44


⇒ –4d = – 20


⇒ d = 5


Putting the value of d in eq. (i), we get


2a + 10×5 = 24


⇒ 2a + 50 = 24


⇒ 2a =24 –50


⇒ 2a =–26


⇒ a = –13


So, the first three terms are –13, –8, –3.



Question 54.

A person was appointed in the pay scale of Rs. 700–40–1500. Find in how many years he will reach maximum of the scale.


Answer:

Let the required number of years = n


Given tn = 1500, a= 700, d = 40


We know that,


tn = a +(n – 1)d


⇒ 1500 = 700 + (n – 1)40


⇒ 800 = (n – 1)40


⇒ 20 = n – 1


⇒ n = 21


Hence, in 21years he will reach maximum of the scale.



Question 55.

A sum of money kept in a hank amounts to Rs. 600/– in 4 years and Rs. 800/– in 12 years. Find the sum and interest carried every year.


Answer:

Let the required sum = a


and the interest carried every year = d


According to question,


In 4years, a sum of money kept in bank account = Rs. 600


i.e. t5 = 600 ⇒ a + 4d = 600 …(i)


and in 12 years , sum of money kept = Rs. 800


i.e. t13 = 800 ⇒ a + 12d = 800 …(ii)


Solving linear equations (i) and (ii), we get


a + 4d – a – 12d = 600 – 800


⇒ – 8d = –200


⇒ d = 25


Putting the value of d in eq.(i), we get


a + 4(25) = 600


⇒ a + 100 = 600


⇒ a = 500


Hence, the sum and interest carried every year is Rs 500 and Rs 25 respectively.



Question 56.

A man starts repaying, a loan with the first instalment of Rs. 100. If he increases the installment by Rs. 5 every month, what amount he will pay in the 30th instalment?


Answer:

The first instalment of the loan = Rs. 100


The 2nd instalment of the loan = Rs. 105


The 3rd instalment of the loan = Rs. 110


and so, on


The amount that the man repays every month forms an AP.


Therefore, the series is


100, 105, 110, 115,…


Here, a = 100, d = 105 – 100 = 5


We know that,


an = a + (n – 1)d


a30 = 100 + (30 – 1)5


⇒ a30 = 100 + 29 × 5


⇒ a30 = 100 +145


⇒ a30 = 245


Hence, the amount he will pay in the 30th installment is Rs 245.




Exercise 8.3
Question 1.

Three numbers are in A.P. Their sum is 27 and the sum of their squares is 275. Find the numbers.


Answer:

Let the three numbers are in AP = a, a + d, a + 2d


According to the question,


The sum of three terms = 27


⇒ a + (a + d) + (a + 2d) = 27


⇒ 3a + 3d = 27


⇒ a + d = 9


⇒ a = 9 – d …(i)


and the sum of their squares = 275


⇒ a2 + (a + d)2 + (a + 2d)2 = 275


⇒ (9 – d)2 + (9)2 + ( 9 – d + 2d)2 = 275 [from(i)]


⇒ 81 + d2 – 18d + 81 + 81 + d2 + 18d = 275


⇒ 243 + 2d2 = 275


⇒ 2d2 = 275 – 243


⇒ 2d2 = 32


⇒ d2 = 16


⇒ d = √16


⇒ d = ±4


Now, if d = 4, then a = 9 – 4 = 5


and if d = – 4, then a = 9 – ( – 4) = 9 + 4 = 13


So, the numbers are →


if a = 5 and d = 4


5, 9, 13


and if a = 13 and d = – 4


13, 9, 5



Question 2.

The sum of three numbers in A.P. is 12 and the sum of their cubes is 408. Find the numbers.


Answer:

Let the three numbers are in AP = a, a + d, a + 2d


According to the question,


The sum of three terms = 12


⇒ a + (a + d) + (a + 2d) = 12


⇒ 3a + 3d = 12


⇒ a + d = 4


⇒ a = 4 – d …(i)


and the sum of their cubes = 408


⇒ a3 + (a + d)3 + (a + 2d)3 = 408


⇒ (4 – d)3 + (4)3 + ( 4 – d + 2d)3 = 408 [from(i)]


⇒ (4 – d)3 + (4)3 + ( 4 + d)3 = 408


⇒ 64 – d3 + 12d2 – 48d + 64 + 64 + d3 + 12d2 + 48d = 408


⇒ 192 + 24d2 = 408


⇒ 24d2 = 408 – 192


⇒ 24d2 = 216


⇒ d2 = 9


⇒ d = √9


⇒ d = ±3


Now, if d = 3, then a = 4 – 3 = 1


and if d = – 3, then a = 4 – ( – 3) = 4 + 3 = 7


So, the numbers are →


if a = 1 and d = 3


1, 4, 7


and if a = 7 and d = – 3


7, 4, 1



Question 3.

Divide 15 into three parts which are in A.P. and the sum of their squares is 83.


Answer:

Let the middle term = a and the common difference = d


The first term = a – d and the succeeding term = a + d


So, the three parts are a – d, a, a + d


According to the question,


Sum of these three parts = 15


⇒ a – d + a + a + d = 15


⇒ 3a = 15


⇒ a = 5


and the sum of their squares = 83


⇒ (a – d)2 + a2 + (a + d)2 = 83


⇒ (5 – d)2 + (5)2 + ( 5 + d)2 = 83 [from(i)]


⇒ 25 + d2 – 10d + 25 + 25 + d2 + 10d = 83


⇒ 75 + 2d2 = 83


⇒ 2d2 = 83 – 75


⇒ 2d2 = 8


⇒ d2 = 4


⇒ d = √4


⇒ d = ±2


Case: (i) If d = 2, then


a – d = 5 – 2 = 3


a = 5


a + d = 5 + 2 = 7


Hence, the three parts are


3, 5, 7


Case: (ii) If d = – 2, then


a – d = 5 – ( – 2) = 7


a = 5


a + d = 5 + ( – 2) = 3


Hence, the three parts are


7, 5, 3



Question 4.

Divide 20 into four parts which are in A.P. such that the ratio of the product of the first and fourth is to the product of the second and third is 2 : 3.


Answer:

Let the four parts which are in AP are


(a – 3d), (a – d), (a + d), (a + 3d)


According to question,


The sum of these four parts = 20


⇒(a – 3d) + (a – d) + (a + d) + (a + 3d) = 20


⇒ 4a = 20


⇒ a = 5 …(i)


Now, it is also given that


product of the first and fourth : product of the second and third = 2 : 3


i.e. (a – 3d) × (a + 3d) : (a – d) × (a + d) = 2 : 3



[∵(a – b)(a + b) = a2 – b2 ]


⇒ 3(a2 – 9d2) = 2(a2 – d2)


⇒ 3a2 – 27d2 = 2a2 – 2d2


⇒ 3a2 – 2a2 = – 2d2 + 27d2


⇒ (5)2 = – 2d2 + 27d2 [from (i)]


⇒ 25 = 25d2


⇒ 1 = d2


⇒ d = ±1


Case I: if d = 1 and a = 5


a – 3d = 5 – 3(1) = 5 – 3 = 2


a – d = 5 – 1 = 4


a + d = 5 + 1 = 6


a + 3d = 5 + 3(1) = 5 + 3 = 8


Hence, the four parts are


2, 4, 6, 8


Case II: if d = – 1 and a = 5


a – 3d = 5 – 3( – 1) = 5 + 3 = 8


a – d = 5 – ( – 1) = 5 + 1 = 6


a + d = 5 + ( – 1) = 5 – 1 = 4


a + 3d = 5 + 3( – 1) = 5 – 3 = 2


Hence, the four parts are


8, 4, 6, 2



Question 5.

Sum of three numbers in A.P. is 21 and their product is 231. Find the numbers.


Answer:

Let the three numbers are (a – d), a and (a + d)


According to question,


Sum of these three numbers = 21


⇒ a – d + a + a + d = 21


⇒ 3a = 21


⇒ a = 7 …(i)


and it is also given that


Product of these numbers = 231


⇒(a – d) × a × (a + d) = 231


⇒(7 – d) × 7 × (7 + d) = 231


⇒ 7 × (72 – d2) = 231 [∵ (a – b)(a + b) = a2 – b2]


⇒ 7 × (49 – d2) = 231


⇒ 343 – 7d2 = 231


⇒ – 7d2 = 231 – 343


⇒ – 7d2 = – 112


⇒ d2 = 16


⇒ d = √16


⇒ d = ±4


Case I: If d = 4 and a = 7


a – d = 7 – 4 = 3


a = 7


a + d = 7 + 4 = 11


So, the numbers are


3, 7, 11


Case II: If d = – 4 and a = 7


a – d = 7 – ( – 4) = 7 + 4 = 11


a = 7


a + d = 7 + ( – 4) = 7 – 4 = 3


So, the numbers are


11, 7, 3



Question 6.

Sum of three numbers in A.P. is 3 and their product is — 35. Find the numbers.


Answer:

Let the three numbers are (a – d), a and (a + d)


According to question,


Sum of these three numbers = 3


⇒ a – d + a + a + d = 3


⇒ 3a = 3


⇒ a = 1 …(i)


and it is also given that


Product of these numbers = – 35


⇒(a – d) × a × (a + d) = – 35


⇒(1 – d) × 1 × (1 + d) = – 35


⇒ 1 × (12 – d2) = – 35 [∵ (a – b)(a + b) = a2 – b2]


⇒ 1 × (1 – d2) = – 35


⇒ 1 – d2 = – 35


⇒ – d2 = – 35 – 1


⇒ – d2 = – 36


⇒ d2 = 36


⇒ d = √36


⇒ d = ±6


Case I: If d = 6 and a = 1


a – d = 1 – 6 = – 5


a = 1


a + d = 1 + 6 = 7


So, the numbers are


– 5, 1, 7


Case II: If d = – 6 and a = 1


a – d = 1 – ( – 6) = 1 + 6 = 7


a = 1


a + d = 1 + ( – 6) = 1 – 6 = – 5


So, the numbers are


7, 1, – 5



Question 7.

If are in A.P. and a + b + c ≠ 0, prove that are in A.P.


Answer:

Given: a + b + c ≠ 0


and are in AP


To Prove: are in AP


if are in AP


[multiplying each term by a + b + c]


i.e. if are in AP


which is given to be true


Hence, are in AP



Question 8.

If a2, b2, c2 are in A.P., show that are in A.P.


Answer:

a2, b2, c2 are in AP


∴ b2 – a2 = c2 – b2


⇒(b – a)(b + a) = (c – b)(c + b)












Hence Proved



Question 9.

If a, b, c are in A.P., prove that

are in A.P.


Answer:

Given: a, b, c are in AP


∴ b – a = c – b …(i)


To Prove: are in AP






⇒ b – a = c – b


∴a, b, c are in AP


are in AP



Question 10.

If a, b, c are in A.P., prove that

(b + c)2 — a2, (c + a)2 — b2, (a + b)2 — c2 are in A.P.


Answer:

Given: a, b, c are in AP


Since, a, b, c are in AP, we have a + c = 2b …(i)


Now, (b + c)2 – a2, (c + a)2 – b2, (a + b)2 – c2 will be in A.P


If (b + c – a)(b + c + a), (c + a – b)(c + a + b), (a + b – c)(a + b + c) are in AP


i.e. if b + c – a, c + a – b, a + b – c are in AP


[dividing by (a + b + c)]


if (b + c – a) + (a + b – c) = 2(c + a – b)


if 2b = 2(c + a – b)


if b = c + a – b


if a + c = 2b which is true by (i)


Hence, (b + c)2 – a2, (c + a)2 – b2, (a + b)2 – c2 are in A.P



Question 11.

If a, b, c are in A.P., prove that

are in A.P.


Answer:

Given: a, b, c are in AP


Since, a, b, c are in AP, we have a + c = 2b …(i)


To Prove : are in AP





⇒ (√b – √a)(√b + √a) = (√c – √b)(√c + √b)


⇒ b – a = c – b


⇒ 2b = a + c, which is True ... from (i)


Hence, the result.



Question 12.

are in A.P., show that are in A.P. provided a + b + c 0


Answer:

Given: are in AP





Taking LCM


⇒b2c + c2b + a2b + ab2 – 2ac2 – 2a2c = 0


⇒ b2c + c2b + a2b + ab2 –ac2 – ac2 – a2c – a2c = 0


⇒(b2c – a2c) + (c2b – ac2) + (a2b – a2c) + (ab2 – ac2) = 0


⇒ c (b – a)(b + a) + c2(b – a) + a2 (b – c) + a(b + c)(b – c) = 0


⇒ c(b – a) {(b + a) + c} + a(b – c) {a + (b + c)} = 0


⇒ (a + b + c){cb – ca + ab – ca} = 0


Given a + b + c ≠ 0


⇒cb – ca + ab – ca = 0


⇒cb – 2ca + ab = 0




are in AP


Hence Proved



Question 13.

If (b – c)2, (c – a)2, (a – b)2 are in A.P., then show that: are in A.P.

[Hint: Add ab + bc + ca — a2 — b2 — c2 to each term or let = b — c, = c— a, = a — b, then + + = 0]


Answer:

Given: (b – c)2, (c – a)2, (a – b)2 are in A.P


∴ 2(c – a)2 = (b – c)2 + (a – b)2 …(i)


To Prove: are in AP


or




⇒2(b – c)(a – b) = (a – c)(c – a)


⇒2[ab – b2 – ca + cb] = ac – a2 – c2 + ac


⇒2ab – 2b2 – 2ac + 2cb = 2ac – a2 – c2


⇒ a2 + c2 – 4ac = 2b2 – 2ab – 2cb


Adding both sides, a2 + c2, we get


⇒2(a2 + c2) – 4ac = a2 + b2 – 2ab + c2 + b2– 2cb


⇒ 2 (a – c)2 = ( b – a)2 + (b – c)2 which is true from (i)


∴(b – c)2, (c – a)2, (a – b)2 are in A.P


are in AP


Hence Proved



Question 14.

If a, b, c are in A.P., prove that:

(a — c)2 = 4 (a — b)(b — c)


Answer:

Given: a, b, c are in AP


∴ a + c = 2b


…(i)


Now taking RHS i.e. 4(a – b)(b – c)


[from(i)]




⇒(a – c)2


= LHS


Hence Proved



Question 15.

If a, b, c are in A.P., prove that:

a3 + c3 + 6abc = 8b3


Answer:

Given: a, b, c are in AP


∴ a + c = 2b …(i)


…(ii)


Taking Lhs i.e. a3 + c3 + 6abc


[from (i)]


⇒ a3 + c3 + 3ac(a + c)


⇒ a3 + c3 3a2c + 3ac2


⇒ (a + c)3


⇒ (2b)3 [from (ii)]


= 8b3 = RHS


Hence Proved



Question 16.

If a, b, c are in A.P., prove that:

(a + 2b — c)(2b + c — a)(c + a — b) = 4abc

[Hint: Put b = on L.H.S. and R.H.S.]


Answer:

Given: a, b, c are in AP


∴ a + c = 2b …(i)


…(ii)


Now, taking LHS i.e. (a + 2b — c)(2b + c — a)(c + a — b)



[from (ii)]




⇒ 4abc


[from (ii)]


= RHS


Hence Proved




Exercise 8.4
Question 1.

The sum of n terms of an A.P. is . Find its 20th term.


Answer:


Taking n = 1, we get




⇒ S1 = 4


⇒ a1 = 4


Taking n = 2, we get




⇒ S2 = 13


∴ a2 = S2 – S1 = 13 – 4 = 9


Taking n = 3, we get




⇒ S3 = 27


∴ a3 = S3 – S2 = 27 – 13 = 14


So, a = 4,


d = a2 – a1 = 9 – 4 = 5


Now, we have to find the 20th term


an = a + (n – 1)d


a20 = 4 + (20 – 1)5


a20 = 4 + 19 × 5


a20 = 4 + 95


a20 = 99


Hence, the 20th term is 99.



Question 2.

The sum of first n terms of an A.P. is given by Sn = 3n2 + 2n. Determine the A.P. and its 15th term.


Answer:

Sn = 3n2 + 2n


Taking n = 1, we get


S1 = 3(1)2 + 2(1)


⇒ S1 = 3 + 2


⇒ S1 = 5


⇒ a1 = 5


Taking n = 2, we get


S2 = 3(2)2 + 2(2)


⇒ S2 = 12 + 4


⇒ S2 = 16


∴ a2 = S2 – S1 = 16 – 5 = 11


Taking n = 3, we get


S3 = 3(3)2 + 2(3)


⇒ S3 = 27 + 6


⇒ S3 = 33


∴ a3 = S3 – S2 = 33 – 16 = 17


So, a = 5,


d = a2 – a1 = 11 – 5 = 6


Now, we have to find the 15th term


an = a + (n – 1)d


a15 = 5 + (15 – 1)6


a15 = 5 + 14 × 6


a15 = 5 + 84


a15 = 89


Hence, the 15th term is 89 and AP is 5, 11, 17, 23,…



Question 3.

The sum of the first n terms of an A.P. is given by Sn = 2n2 + 5n , find the nth term of the A.P.


Answer:

Sn =2n2 + 5n


Taking n = 1, we get


S1 = 2(1)2 + 5(1)


⇒ S1 = 2 + 5


⇒ S1 = 7


⇒ a1 = 7


Taking n = 2, we get


S2 = 2(2)2 + 5(2)


⇒ S2 = 8 + 10


⇒ S2 = 18


∴ a2 = S2 – S1 = 18 – 7 = 11


Taking n = 3, we get


S3 = 2(3)2 + 5(3)


⇒ S3 = 18 + 15


⇒ S3 = 33


∴ a3 = S3 – S2 = 33 – 18 = 15


So, a = 7,


d = a2 – a1 = 11 – 7 = 4


Now, we have to find the 15th term


an = a + (n – 1)d


an = 7 + (n – 1)4


an = 7 + 4n – 4


an = 3 + 4n


Hence, the nth term is 4n + 3.



Question 4.

The sum of n terms of an A.P. is 3n2+ 5n. Find the A.P. Hence, find its 16th term.


Answer:

Sn = 3n2 + 5n


Taking n = 1, we get


S1 = 3(1)2 + 5(1)


⇒ S1 = 3 + 5


⇒ S1 = 8


⇒ a1 = 8


Taking n = 2, we get


S2 = 3(2)2 + 5(2)


⇒ S2 = 12 + 10


⇒ S2 = 22


∴ a2 = S2 – S1 = 22 – 8 = 14


Taking n = 3, we get


S3 = 3(3)2 + 5(3)


⇒ S3 = 27 + 15


⇒ S3 = 42


∴ a3 = S3 – S2 = 42 – 22 = 20


So, a = 8,


d = a2 – a1 = 14 – 8 = 6


Now, we have to find the 15th term


an = a + (n – 1)d


a16 = 8 + (16 – 1)6


a16 = 8 + 15 × 6


a16 = 8 + 90


a16 = 98


Hence, the 16th term is 98.



Question 5.

If the sum of the first n terms of an A.P. is given by Sn = (3n2- n), find its

(i) first term (ii) common difference

(iii) nth term.


Answer:

Sn = 3n2 – n


Taking n = 1, we get


S1 = 3(1)2 - (1)


⇒ S1 = 3 – 1


⇒ S1 = 2


⇒ a1 = 2


Taking n = 2, we get


S2 = 3(2)2 – 2


⇒ S2 = 12 – 2


⇒ S2 = 10


∴ a2 = S2 – S1 = 10 – 2 = 8


Taking n = 3, we get


S3 = 3(3)2 – 3


⇒ S3 = 27 – 3


⇒ S3 = 24


∴ a3 = 24 – 10 = 14


So, a = 1,


d = a2 – a1 = 8 - 2 = 6


Now, we have to find the 15th term


an = a + (n – 1)d


an = 2 + (n – 1)6


an = 2 + 6n – 6


an = - 4 + 6n


Hence, the nth term is 4n - 3.



Question 6.

If the sum to first n terms of an A.P. is , find its 25th term.


Answer:


Taking n = 1, we get




⇒ S1 = 4


⇒ a1 = 4


Taking n = 2, we get




⇒ S2 = 11


∴ a2 = S2 – S1 = 11 – 4 = 7


Taking n = 3, we get




⇒ S3 = 21


∴ a3 = S3 – S2 = 21 – 11 = 10


So, a = 4,


d = a2 – a1 = 7 – 4 = 3


Now, we have to find the 25th term


an = a + (n – 1)d


a25 = 4 + (25 – 1)3


a25 = 4 + 24 × 3


a25 = 4 + 72


a25 = 76


Hence, the 25th term is 76.



Question 7.

If the nth term of an A.P. is (2n + 1), find the sum of first n terms of the A.P.


Answer:

Given: an = 2n + 1


Taking n = 1,


a1 = 2(1) + 1 = 2 + 1 = 3


Taking n = 2,


a2 = 2(2) + 1 = 4 + 1 = 5


Taking n = 3,


a3 = 2(3) + 1 = 6 + 1 = 7


Therefore the series is 3, 5, 7, …


So, a = 3, d = a2 – a1 = 5 – 3 = 2


Now, we have to find the sum of first n terms of the AP






⇒ Sn = 2n + n2


Hence, the sum of n terms is n2 + 2n.



Question 8.

If the nth term of an A.P. is 9 — 5n, find the sum to first 15 terms.


Answer:

Given: an = 9 – 5n


Taking n = 1,


a1 = 9 – 5(1) = 9 – 5 = 4


Taking n = 2,


a2 = 9 – 5(2) = 9 – 10 = -1


Taking n = 3,


a3 = 9 – 5(3) = 9 – 15 = -6


Therefore the series is 4, -1, -6, …


So, a = 4, d = a2 – a1 = -1 – 4 = -5


Now, we have to find the sum of the first 15 terms of the AP






⇒ S15 = 15 × (-31)


⇒ S15 = -465


Hence, the sum of 15 terms is -465.



Question 9.

Find the sum of first 25 terms of an A.P. whose nth term is 1 — 4n.


Answer:

Given: an = 1 – 4n


Taking n = 1,


a1 = 1 – 4(1) = 1 – 4 = -3


Taking n = 2,


a2 = 1 – 4(2) = 1 – 8 = -7


Taking n = 3,


a3 = 1 – 4(3) = 1 – 12 = -11


Therefore the series is -3, -7, -11, …


So, a = -3, d = a2 – a1 = -7 – (-3) = -7 + 3 = -4


Now, we have to find the sum of the first 25 terms of the AP






⇒ S25 = 25 × (-51)


⇒ S25 = -1275


Hence, the sum of 25 terms is -1275.



Question 10.

If the sum to n terms of a sequence be n2 + 2n, then prove that the sequence is an A.P.


Answer:

Given: Sn = n2 + 2n …(i)


Sn-1 = (n – 1)2 + 2(n – 1) = n2 + 1 – 2n + 2n – 2 = n2 - 1 …(ii)


Subtracting eq (ii) from (i), we get


tn = Sn – Sn-1 = n2 + 2n – n2 + 1 = 2n + 1


The nth term of an AP is 2n + 1.



Question 11.

Find the sum to first n terms of an A.P. whose kth term is 5k + 1.


Answer:

As it is given that kth term of the AP = 5k + 1


∴ ak = a + (k – 1)d


⇒ 5k + 1 = a + (k – 1)d


⇒ 5k + 1 = a + kd – d


Now, on comparing the coefficient of k, we get


d = 5


and a – d = 1


⇒ a – 5 = 1


⇒ a = 6


We know that,







Question 12.

If the sum of n terms of an A.P. is 3n2 + 5n and its mth term is 164, find the value of m.

[Hint: tm = Sm — Sm-1= 3m2 + 5m — 3 (m— 1)2 — 5 (m— 1) = 3 (2m — 1) + 5 = 6m + 2]


Answer:

Sn = 3n2 + 5n


Taking n = 1, we get


S1 = 3(1)2 + 5(1)


⇒ S1 = 3 + 5


⇒ S1 = 8


⇒ a1 = 8


Taking n = 2, we get


S2 = 3(2)2 + 5(2)


⇒ S2 = 12 + 10


⇒ S2 = 22


∴ a2 = S2 – S1 = 22 – 8 = 14


Taking n = 3, we get


S3 = 3(3)2 + 5(3)


⇒ S3 = 27 + 15


⇒ S3 = 42


∴ a3 = S3 – S2 = 42 – 22 = 20


So, a = 8,


d = a2 – a1 = 14 – 8 = 6


Now, we have to find the value of m


an = a + (n – 1)d


⇒ am = 8 + (m – 1)6


⇒ 164 = 8 + 6m – 6


⇒ 164 = 2 + 6m


⇒ 162 = 6m


⇒ m = 27



Question 13.

If the sum of n terms of an A.P. is pn + qn2, where p and q are constants, find the common difference.


Answer:

Sn = qn2 + pn


Taking n = 1, we get


S1 = q(1)2 + p(1)


⇒ S1 = q + p


⇒ a1 = q + p


Taking n = 2, we get


S2 = q(2)2 + p(2)


⇒ S2 = 4q + 2p


∴ a2 = S2 – S1 = 4q + 2p – q - p = 3q + p


Taking n = 3, we get


S3 = q(3)2 + p(3)


⇒ S3 = 9q + 3p


∴ a3 = S3 – S2 = 9q + 3p – 4q – 2p = 5q + p


So, a = q + p,


d = a2 – a1 = 3q + p – (q + p) = 3q + p – q – p = 2q


Hence, the common difference is 2q.



Question 14.

If the sum of n terms of an A.P. is nP + 1/2 n( n —1)Q , where P and Q are constants, find the common difference of the A.P.


Answer:


Taking n = 1, we get



⇒ S1 = P


⇒ a1 = P


Taking n = 2, we get



⇒ S2 = 2P + Q


∴ a2 = S2 – S1 = 2P + Q – P = P + Q


Taking n = 3, we get



⇒ S3 = 3P + 3Q


∴ a3 = S3 – S2 = 3P + 3Q – 2P – Q = P + 2Q


So, a = P,


d = a2 – a1 = P + Q – (P) = Q


= a3 – a2 = P + 2Q – (P + Q) = P + 2Q – P – Q = Q


Hence, the common difference is Q.



Question 15.

Find the sum : 25 + 28 + 31 +… + 100


Answer:

Here, a = 25, d = 28 – 25 = 3 and an = 100


We know that,


an = a + (n – 1)d


⇒ 100 = 25 + (n – 1)3


⇒ 75 = (n – 1)3


⇒ 25 = n – 1


⇒ 26 = n


Now,




⇒ S26 = 13[50 + 25 × 3]


⇒ S26 = 13[50 + 75]


⇒ S26 = 13 × 125


⇒ S26 = 1625



Question 16.

Which term of the A.P. 4, 9, 14, ... is 89? Also, find the sum 4 + 9 + 14 + + 89.


Answer:

Let an = 89


AP = 4, 9, 14, …89


Here, a = 4, d = 14 – 9 = 5


We know that


an = a + (n – 1)d


⇒ 89 = 4 + (n – 1)5


⇒ 85 = (n – 1)5


⇒ 17 = n – 1


⇒ 18 = n


So, 89 is the 18th term of the given AP


Now, we find the sum of 4 + 9 + 14 + … + 89


We know that,




⇒ S18 = 9[8 + 17 × 5]


⇒ S18 = 9[8 + 85]


⇒ S18 = 9 × 93


⇒ S18 = 837


Hence, the sum of the given AP is 837.



Question 17.

Solve for x

1 + 6+11 + 16 +...+x= 148


Answer:

Here, a = 1, d = 6 – 1 = 5 and Sn = 148






⇒ 296 = n[5n – 3]


⇒ 5n2 – 3n – 296 = 0


⇒ 5n2 – 40n + 37n – 296 = 0


⇒ 5n(n – 8) + 37(n – 8) = 0


⇒ (5n + 37)(n – 8) = 0


⇒ 5n + 37 = 0 or n – 8 = 0


or n = 8


But is not a positive integer.


∴ n = 8


⇒ x = a8 = a + 7d = 1 + 7 × 5 = 1 + 35 = 36


Hence, x = 36



Question 18.

Solve for x

25+22+19+ 16+...+x= 115


Answer:

Here, a = 25, d = 22 – 25 = -3 and Sn = 115






⇒ 230 = n[53 – 3n]


⇒ 3n2 – 53n + 230 = 0


⇒ 3n2 – 30n - 23n + 230 = 0


⇒ 3n(n – 10) - 23(n – 10) = 0


⇒ (3n – 23)(n – 10) = 0


⇒ 3n – 23 = 0 or n – 10 = 0


or n = 10


But is not an integer.


∴ n = 10


⇒ x = a10 = a + 9d = 25 + 9 × (-3) = 25 – 27 = -2


Hence, x = -2



Question 19.

Find the number of terms of the A.P. 64, 60, 56, ... so that their sum is 544. Explain the double answer.


Answer:

AP = 64, 60, 56, …


Here, a = 64, d = 60 – 64 = -4






⇒ 1088 = n[132 – 4n]


⇒ 4n2 – 132n + 1088 = 0


⇒ n2 – 33n + 272= 0


⇒ n2 – 16n - 17n + 272 = 0


⇒ n(n – 16) - 17(n – 16) = 0


⇒ (n – 16)(n – 17) = 0


⇒ n – 16 = 0 or n – 17 = 0


⇒ n = 16 or n = 17


If n = 16, a = 64 and d = -4


a16 = 64 + (16 – 1)(-4)


a16 = 64 + 15 × -4


a16 = 64 – 60


a16 = 4


and If n = 17, a = 64 and d = -4


a17 = 64 + (17 – 1)(-4)


a17 = 64 + 16 × -4


a17 = 64 – 64


a17 = 0


Now, we will check at which term the sum of the AP is 544.




⇒ S16 = 8[68]


⇒ S16 = 544


and


⇒ S17 = 17 × 32


⇒ S17 = 544


So, the terms may be either 17 or 16 both holds true.


We get a double answer because the 17th term is zero and when we add this in the sum, the sum remains the same.



Question 20.

How many terms of the A.P. 3, 5, 7, 9, ... must be added to get the sum 120?


Answer:

AP = 3, 5, 7, 9, …


Here, a = 3, d = 5 – 3 = 2 and Sn = 120


We know that,







⇒ 120 = n[2+n]


⇒ n2 + 2n – 120 = 0


⇒ n2 + 12n – 10n – 120 = 0


⇒ n(n + 12) - 10(n + 12) = 0


⇒ (n – 10)(n + 12) = 0


⇒ n – 10 = 0 or n + 12 = 0


⇒ n = 10 or n = -12


But number of terms can’t be negative. So, n = 10


Hence, for n = 10 the sum is 120 for the given AP.



Question 21.

Find the number of terms of the A.P. 63, 60, 57, ... so that their sum is 693. Explain the double answer.


Answer:

AP = 63, 60, 57,…


Here, a = 63, d = 60 – 63 = -3 and Sn = 693


We know that,







⇒ 1386 = n[129 – 3n]


⇒ 3n2 – 129n + 1386 = 0


⇒ n2 – 43n + 462 = 0


⇒ n2 – 22n – 21n + 462 = 0


⇒ n(n – 22) - 21(n – 22) = 0


⇒ (n – 21)(n – 22) = 0


⇒ n – 21 = 0 or n – 22 = 0


⇒ n = 21 or n = 22


So, n = 21 and 22


If n = 21, a = 63 and d = -3


a21 = 63 + (21 – 1)(-3)


a21 = 63 + 20 × -3


a21 = 63 – 60


a21 = 3


and If n = 22, a = 63 and d = -3


a22 = 63 + (22 – 1)(-3)


a22 = 63 + 21 × -3


a22 = 63 – 63


a22 = 0


Now, we will check at which term the sum of the AP is 693.




⇒ S21 = 21 × 33


⇒ S21 = 693


and


⇒ S22 = 11 × 63


⇒ S22 = 693


So, the terms may be either 21 or 22 both holds true.


We get the double answer because here the 22nd term is zero and it does not affect the sum.



Question 22.

How many terms of the series 15 + 12 + 9 + ... must be taken to make 15? Explain the double answer.


Answer:

Here, a = 15, d = 12 – 15 = -3 and Sn = 15


We know that,







⇒ 30 = n[33 – 3n ]


⇒ 3n2 – 33n + 30 = 0


⇒ 3n2 – 30n – 3n + 30 = 0


⇒ 3n(n – 10) -3(n – 10) = 0


⇒ (n – 10)(3n – 3) = 0


⇒ n – 10 = 0 or 3n – 3 = 0


⇒ n = 10 or n = 1


The number of terms can be 1 or 10.


Here, the common difference is negative.


∴ The AP starts from a positive term, and its terms are decreasing.


∴ All the terms after 6th term are negative.


We get a double answer because these positive terms from 2nd to 5th term when added to negative terms from 7th to 10th term, they cancel out each other and the sum remains same.



Question 23.

Find the sum of all the odd numbers lying between 100 and 200.


Answer:

The odd numbers lying between 100 and 200 are


101, 103, 105,…, 199


a2 – a1 = 103 – 101 = 2


a3 – a2 = 105 – 103 = 2


∵ a3 – a2 = a2 – a1 = 2


Therefore, the series is in AP


Here, a = 101, d = 2 and an = 199


We know that,


an = a + (n – 1)d


⇒ 199 = 101 + (n – 1)2


⇒ 199 – 101 = (n – 1)2


⇒ 98 = (n – 1)2


⇒ 49 = (n – 1)


⇒ n = 50


Now, we have to find the sum of this AP




⇒ S50 = 25[202 + 49 × 2]


⇒ S50 = 25[300]


⇒ S50 = 7500


Hence, the sum of all odd numbers lying between 100 and 200 is 7500.



Question 24.

Find the sum of all odd integers from 1 to 2001.


Answer:

The odd numbers lying between 1 and 2001 are


1, 3, 5,…, 2001


a2 – a1 = 3 – 1 = 2


a3 – a2 = 5 – 3 = 2


∵ a3 – a2 = a2 – a1 = 2


Therefore, the series is in AP


Here, a = 1, d = 2 and an = 2001


We know that,


an = a + (n – 1)d


⇒ 2001 = 1 + (n – 1)2


⇒ 2001 – 1 = (n – 1)2


⇒ 2000 = (n – 1)2


⇒ 1000 = (n – 1)


⇒ n = 1001


Now, we have to find the sum of this AP




⇒ S1001 = 1001[1 + 1000]


⇒ S1001 = 1001 [1001]


⇒ S1001 = 1002001


Hence, the sum of all odd numbers lying between 1 and 2001 is 1002001.



Question 25.

Determine the sum of first 35 terms of an A.P., if the second term is 2 and the seventh term is 22.


Answer:

Given: a2 = 2 and a7 = 22 and n = 35


We know that,


a2 = a + d = 2 …(i)


and a7 = a + 6d = 22 …(ii)


Solving the linear equations (i) and (ii), we get


a + d – a – 6d = 2 – 22


⇒ - 5d = -20


⇒ d =4


Putting the value of d in eq. (i), we get


a + 4 = 2


⇒ a = 2 – 4 = -2


Now, we have to find the sum of first 35 terms.





⇒ S35 = 35 [-2 + 34 × 2]


⇒ S35 = 35 [66]


⇒ S35 = 2310



Question 26.

If the sum of the first p terms of an A.P. is q and the sum of first q terms is p, then find the sum of first (p + q) terms.


Answer:

Given: Sp = q and Sq = p


To find: Sp+q


We know that,






…(i)


Now,





…(ii)


From eq. (i) and (ii), we get






[∵, a2 – b2 = (a – b)(a + b)]



…(iii)


Now, putting the value of d in eq. (i), we get




…(iv)


Now, we to find Sp+q




[from (iii) & (iv)]






⇒ Sp+q = - (p+q)


Hence, the sum of first (p+q) terms is –(p + q)



Question 27.

How many terms of the A.P. -6,-,-5 ... are needed to get the sum - 25?


Answer:


Here, a = -6,



and Sn = -25


We know that,







⇒ -100 = n[-25 + n]


⇒ n2 – 25n + 100 = 0


⇒ n2 – 20n – 5n + 100 = 0


⇒ n(n – 20) - 5(n – 20) = 0


⇒ (n – 20)(n – 5) = 0


⇒ n – 5 = 0 or n – 20 = 0


⇒ n = 5 or n = 20


So, n = 5 or 20



Question 28.

Find the sum of the numbers lying between 107 and 253 that are multiples of 5.


Answer:

The numbers lying between 107 and 253 that are multiples of 5 are


110, 115, 120,…, 250


a2 – a1 = 115 – 110 = 5


a3 – a2 = 120 – 115 = 5


∵ a3 – a2 = a2 – a1 = 5


Therefore, the series is in AP


Here, a = 110, d = 5 and an = 250


We know that,


an = a + (n – 1)d


⇒ 250 = 110 + (n – 1)5


⇒ 250 – 110 = (n – 1)5


⇒ 140 = (n – 1)5


⇒ 28 = (n – 1)


⇒ n = 29


Now, we have to find the sum of this AP




⇒ S29 = 29[110 + 14 × 5]


⇒ S29 = 29[180]


⇒ S29 = 5220


Hence, the sum of all numbers lying between 107 and 253 is 5220.



Question 29.

Find the sum of all natural numbers lying between 100 and 1000 which are multiples of 5.


Answer:

The numbers lying between 100 and 1000 that are multiples of 5 are


105, 110, 115, 120,…, 995


a2 – a1 = 110 – 105 = 5


a3 – a2 = 115 – 110 = 5


∵ a3 – a2 = a2 – a1 = 5


Therefore, the series is in AP


Here, a = 105, d = 5 and an = 995


We know that,


an = a + (n – 1)d


⇒ 995 = 105 + (n – 1)5


⇒ 995 – 105 = (n – 1)5


⇒ 890 = (n – 1)5


⇒ 178 = (n – 1)


⇒ n = 179


Now, we have to find the sum of this AP




⇒ S179 = 179[105 + 89 × 5]


⇒ S179 = 179 [550]


⇒ S179 = 98450


Hence, the sum of all numbers lying between 100 and 1000 that are multiples of 5 is 98450.



Question 30.

Find the sum of all the two digit odd positive integers.


Answer:

The two digit odd positive integers are


11, 13, 15,…, 99


a2 – a1 = 13 – 11 = 2


a3 – a2 = 15 – 13 = 2


∵ a3 – a2 = a2 – a1 = 2


Therefore, the series is in AP


Here, a = 11, d = 2 and an = 99


We know that,


an = a + (n – 1)d


⇒ 99 = 11 + (n – 1)2


⇒ 99 – 11 = (n – 1)2


⇒ 88 = (n – 1)2


⇒ 44 = (n – 1)


⇒ n = 45


Now, we have to find the sum of this AP




⇒ S45 = 45[11 + 44]


⇒ S45 = 45[55]


⇒ S45 = 2475


Hence, the sum of all two digit odd numbers are 2475.



Question 31.

Find the sum of all multiplies of 9 lying between 300 and 700.


Answer:

The numbers lying between 300 and 700 which are multiples of 9 are


306, 315, 324,…, 693


a2 – a1 = 315 – 306 = 9


a3 – a2 = 324 – 315 = 9


∵ a3 – a2 = a2 – a1 = 9


Therefore, the series is in AP


Here, a = 306, d = 9 and an = 693


We know that,


an = a + (n – 1)d


⇒ 693 = 306 + (n – 1)9


⇒ 693 - 306 = (n – 1)9


⇒ 387 = (n – 1)9


⇒ 43 = (n – 1)


⇒ n = 44


Now, we have to find the sum of this AP




⇒ S44 = 22[612 + 387]


⇒ S44 = 22[999]


⇒ S44 = 21978


Hence, the sum of all numbers lying between 300 and 700 is 21978.



Question 32.

Find the sum of all the three digit natural numbers which are multiples of 7.


Answer:

The three digit natural numbers which are multiples of 7 are


105, 112, 119,…, 994


a2 – a1 = 112 – 105 = 7


a3 – a2 = 112 – 105 = 7


∵ a3 – a2 = a2 – a1 = 7


Therefore, the series is in AP


Here, a = 105, d = 7 and an = 994


We know that,


an = a + (n – 1)d


⇒ 994 = 105 + (n – 1)7


⇒ 994 – 105 = (n – 1)7


⇒ 889 = (n – 1)7


⇒ 127 = (n – 1)


⇒ n = 128


Now, we have to find the sum of this AP




⇒ S128 = 64[210 + 127 × 7]


⇒ S128 = 64[1099]


⇒ S128 = 70336


Hence, the sum of all three digit numbers which are multiples of 7 are 70336.



Question 33.

Find the sum of all natural numbers lying between 100 and 500, which are divisible by 8.


Answer:

The numbers lying between 100 and 500 which are divisible by 8 are


104, 112, 120, 128, 136,…, 496


a2 – a1 = 112 – 104 = 8


a3 – a2 = 120 – 112 = 8


∵ a3 – a2 = a2 – a1 = 8


Therefore, the series is in AP


Here, a = 120, d = 8 and an = 496


We know that,


an = a + (n – 1)d


⇒ 496 = 104 + (n – 1)8


⇒ 496 – 104 = (n – 1)8


⇒ 392 = (n – 1)8


⇒ 49 = (n – 1)


⇒ n = 50


Now, we have to find the sum of this AP




⇒ S50 = 25[208 + 49 × 8]


⇒ S50 = 25[600]


⇒ S50 = 15000


Hence, the sum of all numbers lying between 100 and 500 and divisible by 8 is 15000.



Question 34.

Find the sum of all the 3 digit natural numbers which are divisible by 13.


Answer:

The three digit natural numbers which are divisible by 13 are


104, 117, 130,…, 988


a2 – a1 = 117 – 104 = 13


a3 – a2 = 130 – 117 = 13


∵ a3 – a2 = a2 – a1 = 13


Therefore, the series is in AP


Here, a = 104, d = 13 and an = 988


We know that,


an = a + (n – 1)d


⇒ 988 = 104 + (n – 1)13


⇒ 988 – 104 = (n – 1)13


⇒ 884 = (n – 1)13


⇒ 68 = (n – 1)


⇒ n = 69


Now, we have to find the sum of this AP





⇒ S69 = 69[104 + 34 × 13]


⇒ S69 = 69[546]


⇒ S69 = 37674


Hence, the sum of three digit natural numbers which are divisible by 13 are 37674.



Question 35.

The 5th and 15th terms of an A.P. are 13 and - 17 respectively. Find the sum of first 21 terms of the A.P.


Answer:

Given: a5 = 13 and a15 = -17 and n = 21


We know that,


a5 = a + 4d = 13 …(i)


and a15 = a + 14d = -17 …(ii)


Solving the linear equations (i) and (ii), we get


a + 4d – a – 14d = 13 – (-17)


⇒ -10d = 13 + 17


⇒ -10d = 30


⇒ d = -3


Putting the value of d in eq. (i), we get


a + 4(-3) = 13


⇒ a = 13 + 12 = 25


Now, we have to find the sum of first 21 terms.





⇒ S21 = 21 [25 + 10 × (-3)]


⇒ S21 = 21 [-5]


⇒ S21 =-105



Question 36.

Find the sum of first 21 terms of the A.P. whose 2nd term is 8 and 4th term is 14.


Answer:

Given: a2 = 8 and a4 = 14 and n = 21


We know that,


a2 = a + d = 8 …(i)


and a4 = a + 3d = 14 …(ii)


Solving the linear equations (i) and (ii), we get


a + d – a – 3d = 8 – 14


⇒ -2d = -6


⇒ d = 3


Putting the value of d in eq. (i), we get


a + 3 = 8


⇒ a = 8 – 3 = 5


Now, we have to find the sum of first 21 terms.





⇒ S21 = 21 [5 + 10 × (3)]


⇒ S21 = 21 [35]


⇒ S21 = 735



Question 37.

Find the sum of 51 terms of the A.P. whose second term is 2 and the 4th term is 8.


Answer:

Given: a2 = 2 and a4 = 8 and n = 51


We know that,


a2 = a + d = 2 …(i)


and a4 = a + 3d = 8 …(ii)


Solving the linear equations (i) and (ii), we get


a + d – a – 3d = 2 – 8


⇒ -2d = -6


⇒ d = 3


Putting the value of d in eq. (i), we get


a + 3 = 2


⇒ a = 2 – 3 = -1


Now, we have to find the sum of first 51 terms.





⇒ S51 = 51 [-1 + 25 × (3)]


⇒ S51 = 51 [74]


⇒ S51 = 3774



Question 38.

Find the sum of the first 25 terms of the A.P. whose 2nd term is 9 and 4th term is 21.


Answer:

Given: a2 = 9 and a4 = 21 and n = 25


We know that,


a2 = a + d = 9 …(i)


and a4 = a + 3d = 21 …(ii)


Solving the linear equations (i) and (ii), we get


a + d – a – 3d = 9 – 21


⇒ -2d = -12


⇒ d = 6


Putting the value of d in eq. (i), we get


a + 6 = 9


⇒ a = 9 – 6 = 3


Now, we have to find the sum of first 25 terms.





⇒ S25 = 25 [3 + 12 × (6)]


⇒ S25 = 25 [75]


⇒ S25 = 1875



Question 39.

If the sum of 8 terms of an A.P. is 64 and the sum of 19 terms is 361, find the sum of n terms.


Answer:

Given: S8 = 64 and S19 = 361


We know that,




⇒ 64 = 4 [2a +7d]


⇒ 16 = 2a + 7d …(i)


Now,




⇒ 38 = 2a + 18d …(ii)


Solving linear equations (i) and (ii), we get


2a + 7d – 2a – 18d = 16 – 38


⇒ -11d = -22


⇒ d = 2 …(iii)


Putting the value of d in eq. (i), we get


2a + 7(2) = 16


⇒ 2a = 16 – 14


⇒ 2a = 2 …(iv)


Now, we have to find the Sn



[from (iii) and (iv)]


⇒ Sn = n [1 + n – 1]


⇒ Sn = n2



Question 40.

The first and the last terms of an A.P. are 17 and 350 respectively. If the common difference is 9, how many terms are there in the A.P. and what is their sum?


Answer:

Given: First term, a = 17


Last term, l = 350


common difference, d = 9


We know that,


l = a + (n – 1)d


⇒ 350 = 17 + (n – 1)9


⇒ 333 = (n – 1)9


⇒ 37 = n – 1


⇒ n = 38


So, there are 38 terms in the AP


Now, we have to find the sum of this AP




⇒ S38 = 19 [34 +37×9]


⇒ S38 = 19 [34 + 333]


⇒ S38 = 19 × 367


⇒ S38 = 6973


Hence, the sum of 38 terms is 6973.



Question 41.

If a, b, c be the 1st, 3rd and nth terms respectively of an A.P., there prove that the sum to n terms is


Answer:

Given: a1 = a


a3 = a + 2d = b


⇒ 2d = b – a



and an = a + (n – 1)d








We know that,









Question 42.

If the mth term of an A.P. is and the nth term is , then prove that the sum to mn terms is , where in m n.


Answer:

Given:


Now, am = a + (m – 1)d



⇒ an + n(m – 1)d = 1


⇒ an + mnd – nd = 1 …(i)




⇒ am + mnd – md = 1 …(ii)


From eq. (i) and (ii), we get


an + mnd – nd = am + mnd – md


⇒ a(n – m) –d (n – m) = 0


⇒ a = d


Now, putting the value of a in eq. (i), we get


dn + mnd – nd = 1


⇒ mnd = 1



Hence,


Sum of mn terms of AP is







Hence Proved



Question 43.

If the 12th term of an A.P. is - 13 and the sum of the first four terms is 24, what is the sum of the first 10 terms?


Answer:

Given: a12 = -13


⇒ a + 11d = -13


⇒ a = -13 – 11d …(i)


and S4 = 24


[from(i)]


⇒ 2[-26 -22d + 3d] = 24


⇒ -26 – 19d = 12


⇒ -19d = 12 + 26


⇒ -19d = 38


⇒ d = -2


Putting the value of d in eq. (i), we get


a = -13 – 11(-2) = -13 + 22 = 9


So, a = 9 , d = -2 and n = 10


Now, we have to find the S10



⇒ S10 = 5[2×9 + 9(-2)]


⇒ S10 = 5[18 – 18]


⇒ S10 = 0


Hence, the sum of first 10 terms is 0



Question 44.

If the number of terms of an A.P. be 2n + 3, then find the ratio of sum of the odd terms to the sum of even terms.


Answer:

Given: Total number of terms = 2n + 3


Let the first term = a


and the common difference = d


Then, ak = a + (k – 1)d …(i)


Let S1 and S2 denote the sum of all odd terms and the sum of all even terms respectively.


Then,


S1 = a1 + a3 + a5 … + a2n+3



[using (i)]



= (n + 2)(a + nd + d) …(ii)


And, S2 = a2 + a4 + a6 … + a2n+2



[using (i)]



= (n+1)(a + nd + d) …(iii)




Question 45.

If the sum of first m terms of an A.P. is the same as the sum of its first n terms, show that the sum of its first (m + n) terms is zero.


Answer:

Let the first term be a and common difference of the given AP is d.


Given: Sm = Sn



⇒ 2am + md(m – 1) = 2an + nd(n – 1)


⇒ 2am – 2an + m2d – md – n2d + nd = 0


⇒ 2a (m – n) + d[(m2 – n2) – (m – n)] = 0


⇒ 2a (m – n) + d[(m– n)(m + n) – (m – n)] = 0


⇒ (m – n) [2a + {(m + n) – 1}d] = 0


⇒ 2a + (m + n – 1)d = 0 [∵ m – n ≠ 0]…(i)


Now,



[using (i)]


⇒ Sm+n = 0


Hence Proved



Question 46.

In an A.P. the first term is 2, and the sum of the first five terms is one-fourth of the next five terms. Show that its 20th term is — 112.


Answer:

Given: first term, a = 2


And


Sum of first five terms


Sum of next 5 terms




⇒ 4S5 = S10 – S5


⇒ 5S5 = S10




⇒ 20 + 20d = 8 + 18d


⇒ 20d – 18d = 8 – 20


⇒ 2d = -12


⇒ d = -6


Thus, a = 2 and d = -6


∴ a20 = a + (n – 1)d


⇒ a20 = 2 + (20 – 1)(-6)


⇒ a20 = 2 + (19)(-6)


⇒ a20 = 2 – 114


⇒ a20 = -112


Hence Proved



Question 47.

If d be the common difference of an A.P. and Sn be the sum of its n terms, then prove that d = Sn - 2Sn-1 + Sn-2


Answer:

Given: Sn be the sum of n terms and d be the common difference.


To Prove: d = Sn - 2Sn-1 + Sn-2


Taking RHS


Sn - 2Sn-1 + Sn-2







= d


=LHS


Hence Proved



Question 48.

The sum of the first 7 terms of an A.P. is 10, and that of the next 7 terms is 17. Find the progression.


Answer:

Given: Sum of first 7 terms, S7 = 10


and Sum of the next 7 terms = 17


⇒ Sum of 8th to 14th terms = 17


⇒ Sum of first 14 terms – Sum of first 7 terms = 17


⇒ S14 – S7 = 17


⇒ S14 – 10 = 17


⇒ S14 = 27


Sum of 7 terms,



⇒ 20 = 7[2a + 6d]


⇒ 20 = 14a + 42d …(i)


Sum of 14 terms,


⇒ 27 = 7[2a + 13d]


⇒ 27 = 14a + 91d …(ii)


Solving the linear equations (i) and (ii), we get


14a + 42d – 14a – 91d = 20 – 27


⇒ -49d = -7



Putting the value of d in eq. (i), we get


20 = 14a + 42d



⇒ 20= 14a + 6


⇒ 20 – 6 = 14a


⇒ 14 = 14a


⇒ a = 1


Thus, a = 1 and


So, AP is


a1 = 1


a2


a3


Hence, AP is



Question 49.

If the pth term of an A.P. is x and qth term is y, show that the sum of (p + q) terms is


Answer:

Given: ap = x and aq = y


We know that,


an = a + (n – 1)d


ap = a + (p – 1)d


⇒ x = a + (p – 1)d …(i)


Now,


aq = a + (q – 1)d


⇒ y = a + (q – 1)d …(ii)


From eq. (i) and (ii), we get


x – (p – 1)d = y – (q – 1)d


⇒ x – y = (p – 1)d – (q – 1)d


⇒ x – y = d [p – 1 – q + 1]


⇒ x – y = d[ p – q]


…(iii)


Adding, Eq (i) and (ii), we get


x + y = 2a + (p – 1) + (q – 1)d


⇒ x + y = 2a + d[p + q – 1 – 1]


⇒ x + y = 2a + d (p + q – 1) –d


⇒ x + y + d = 2a + (p + q – 1)d …(iv)


We know that,




[using (iv)]


[using (iii)]


Hence Proved



Question 50.

The sum of 17 terms of two series in A.P. are in the ratio (3n + 8) : (7n + 15). Find the ratio of their 12th terms.


Answer:

There are two AP with different first term and common difference.


For the First AP


Let first term be a


Common difference = d


Sum of n terms =


and nth term = an = a + (n – 1)d


For the second AP


Let first term be A


Common difference = D


Sum of n terms =


and nth term = An = A + (n – 1)D


It is given that





…(i)


Now, we need to find ratio of their 12th term






Hence,


n – 1 = 11 × 2


⇒ n = 22 + 1


⇒ n = 23


Putting n = 23 in eq. (i), we get






Hence the ratio of 12th term of 1st AP and 12th term if 2nd AP is 7:16



Question 51.

The sum of 11 terms of two A.P.'s are in the ratio (5n + 4) : (9n + 6), find the ratio of their 18th terms.


Answer:

There are two AP with different first term and common difference.


For the First AP


Let first term be a


Common difference = d


Sum of n terms =


and nth term = an = a + (n – 1)d


For the second AP


Let first term be A


Common difference = D


Sum of n terms =


and nth term = An = A + (n – 1)D


It is given that





…(i)


Now, we need to find ratio of their 18th term






Hence,


n – 1 = 17 × 2


⇒ n = 34 + 1


⇒ n = 35


Putting n = 35 in eq. (i), we get





Hence the ratio of 18th term of 1st AP and 18th term if 2nd AP is 179:321



Question 52.

In an A.P. Sn denotes the sum to first n terms, if Sn = n2p and Sm = m2p (m n) prove that Sp = p3.


Answer:

Given: Sn = n2p and Sm = m2p


To Prove: Sp = p3


We know that,




⇒ 2np = [2a + (n – 1)d]


⇒ 2np – (n – 1)d = 2a …(i)


and



⇒ 2mp = 2a + (m – 1)d


⇒ 2mp – (m – 1)d = 2a …(ii)


From eq. (i) and (ii), we get


⇒ 2np – (n – 1)d = 2mp – (m – 1)d


⇒ 2np – nd + d = 2mp – md + d


⇒ 2np – nd = 2mp – md


⇒ md – nd = 2mp – 2np


⇒ d(m – n) = 2p(m – n)


⇒ d = 2p …(iii)


Putting the value of d in eq. (i), we get


⇒ 2np – (n – 1)(2p) = 2a


⇒ 2pn – 2pn + 2p = 2a


⇒ 2p = 2a …(iv)


Now, we have to find the Sp


[from (iii) & (iv)]




⇒ Sp = p3


Hence Proved



Question 53.

The income of a person is Rs. 300000 in the first year and he receives an increase of Rs. 10000 to his income per year for the next 19 years. Find the total amount he received in 20 years.


Answer:

The income of a person in 1st year = Rs 300000


The income of a person in 2nd year = Rs 300000 + 10000


= Rs 310000


The income of a person in 3rd year = Rs 310000 + 10000


= Rs 320000


and so,on


Therefore, the AP is


300000, 310000, 320000,…


Here a = 300000, d = 310000 – 300000 = 10000


and n = 20


We know that,




⇒ S20 = 10 [600000 + 190000]


⇒ S20 = 10[790000]


⇒ S20 = 7900000


Hence, the total amount he received in 20 years is Rs 7900000.



Question 54.

A man starts repaying a loan as first installment of Rs. 100. If he increases the installments by Rs. 5 every month, what amount he will pay in 30 installments?


Answer:

The 1st installment of the loan = Rs. 100


the 2nd installment of the loan = Rs 100 + 5 = Rs 105


The 3rd installment of the loan = Rs 105 + 5 = Rs 110


Therefore, the AP is 100, 105, 110, …


Here, a = 100, d = 105 – 100 = 5 and n = 30


We know that,




⇒ S30 = 15 [200 + 29 × 5]


⇒ S30 = 15 [200+145]


⇒ S30 = 15 [345]


⇒ S30 = 5175


Hence, the amount he will pay in 30th installments is Rs. 5175



Question 55.

The interior angles of a polygon are in A.P., the smallest angle is 75° and the common difference is 10°. Find the number of sides of the polygon.


Answer:

Given: The smallest angle is 75°


i.e. a = 75


and common difference = 10°


i.e. d = 10


Therefore, the series is


75, 85, 95, 105, …


and the sum of interior angles of a polygon =(n – 2) 180°


i.e. Sn = 180


We know that,





⇒ (n – 2)360 = n [140+10n]


⇒ 360n – 720 = 140n + 10n2


⇒ 36n – 72 – 14n – n2 = 0


⇒ n2 – 22n + 72 = 0


⇒ n2 – 18n – 4n + 72 = 0


⇒ n(n – 18) – 4(n – 18) = 0


⇒ (n – 4)(n – 18) = 0


Putting both the factor equal to 0, we get


n – 4 = 0 or n – 18 = 0


⇒ n = 4 or n = 18


Hence, the number of sides of a polygon can be 4 or 18.